Этого треда уже нет.
Это копия, сохраненная 12 сентября 2018 года.

Скачать тред: только с превью, с превью и прикрепленными файлами.
Второй вариант может долго скачиваться. Файлы будут только в живых или недавно утонувших тредах. Подробнее

Если вам полезен архив М.Двача, пожертвуйте на оплату сервера.
Тред тупых вопросов №82 367033 В конец треда | Веб
Тред вопросов о жизни, Вселенной и всем таком.

Спрашиваем то, за что в других местах выдают путёвку в биореактор. Здесь анонимные ученые мирового уровня критически рассмотрят любые гениальные идеи и нарисованные в Paint схемы.

Предыдущий тут: https://2ch.hk/spc/res/363848.html (М)

Q: Можно быстрее?
A: Можно упасть в пузырь альбукерке, наса уже почти надула его.

Q: Я начитался охуительных историй про уфологию, че делать, нам жопа?
A: Да, тебе жопа, можешь сгонять в зогач или куда оттуда пошлют.

Q: Что будет с человеком в вакууме без скафандра / если он упадет на черную дыру / попробует ступить на поверхность газового гиганта/солнца?
A: Он умрёт.
2 367103
>>67033 (OP)

>в других местах выдают путёвку в биореактор


А тут не выдают?
3 367105
>>67103
Если совсем упоротый наркоман с IQ равным размеру обуви, то могут и тут.
4 367116
>>67105
Далеко пойдёте с таким низким порогом приятия
5 367120
какие ещё есть перспективные и не очень способы достижения космического пространства и оставания на орбите?
1. ракетный двигатель, ну тут без вариантов.
2. космический лифт.
3. магнитный дирижабль с взаимодействием магнитым полем земли.
4. детонационные двигатели.
5......?
6 367121
>>67120
солнечный парус, ярд, ионники
7 367123
И вообще, вон НАСА недавно кукарекнуло, их и спроси https://www.newscientist.com/article/mg23631576-000-exclusive-nasa-has-begun-plans-for-a-2069-interstellar-mission/
8 367125
>>67121
с поверхности подходит ядерный.
9 367127
>>67116

>Тред тупых вопросов №82


>№82


>Далеко пойдете.


Не только далеко, еще и на долго. ТТВ тут уже лет пять существует. С 13 года, или 14, я уже не помню.
10 367130
>>67125
Ядерный и в космосе прекрасно работает.
11 367132
>>67130
да не, анон спрашивал про выход на орбиту, я жопой прочитал
Screenshot2017-12-26-06-52-50-811com.mishiranu.dashchan.png217 Кб, 1080x1920
12 367143
>>67123
Ну бля
13 367238
>>67120

>какие ещё есть перспективные и не очень способы достижения космического пространства и оставания на орбите?


>1. ракетный двигатель, ну тут без вариантов.


>2. манягалиматья аутиста


>3. манягалиматья довена


>4. детсадовская манягалиматья


>5......Лазер


>6......Лазер


>7......Лазер


Пачинил
14 367308
Спэйсач резиновый что ли? До вайпа было восемь страниц, теперь тринадцать.
sage 15 367322
>>67308
Дващ вайпать бесполезно жеж. На время вайпа разделы резиновыми становятся, это давно замечено. Сколько угодно тредов влезет, и нижние не смываются, пока шухер не уляжется.
16 367328
>>67322

>Дващ вайпать бесполезно жеж.


>Сколько угодно тредов влезет, и нижние не смываются


Вот только у меня половина избранного улетело к хуям.
17 367329
Щито? Кто-то спейсач вайпнул?
18 367331
>>67329
Да тут весь харкач какой-то конфочкой на нуль делили.
19 367332
>>67329
Ты и конец света проспишь.
20 367333
>>67329
Плоскоземельщики
21 367334
>>67331
Как это выглядело-то?

>>67332
Да, сдаю позиции. Больше не капчую 24/7.
Иногда захожу на спейсач обсудить омск, посмотреть пуск и поотвечать в ТТВ или побампать его своими тупыми вопросами на которые и так знаю ответ
22 367335
>>67328
В этом разделе все на месте. Тред про гравиволны как вчера болтался последним, так и сейчас там болтается. Про другие разделы не знаю. По идее ничего улететь не должно было. Может сама функция избранного проглючила, не?
Закладывать в избранное некротреды, хех
23 367337
>>67335

>В этом разделе все на месте.


Разве что до третьей страницы. Избранное куклы, если что. Каждый помеченый как 404 прочекал — нот фаунд.
>спойлер
Я их оттуда не убираю. Мне нравится наблюдать, как они потихоньку тонут.
24 367339
>>67337
Тред про жуков навозников на месте, я спокоен.
Fuck This Shit Im Out.webm986 Кб, webm,
640x360, 0:24
25 367349
Если тред про Gaia наебнется, то я уебываю.
26 367352
>>67033 (OP)
Что на оппике делает худой Кевин Спейси?
27 367353
>>67352
Открывает тела в поясе Койпера с какой-то пиздоглазой вьетнамкой.
28 367373
>>67120

>космический лифт


>оставания на орбите


Только на оче высокой если. На НОО с лифтом без движков один хуй никуда.
29 367382
>>67349
а) он остался жив и висит сейчас последним
б) вайп уже прекратился давно
30 367385
>>67335
Смешные картинки улетели
31 367388
>>67385
Хвала Аллаху и мудрости его.
32 367396
>>67385
Он и так за бамплимитом был уже давно, в архиваче можно помотреть спокойно. А вот второму космофактотреду пиздец, обидно inb4 всем похуй кроме ОПа.
Надеюсь у этого вайпера-конфочкопетуха будет самый болезненный рак яиц в истории Вселенной.
33 367397
>>67396
В архиваче его нет?
34 367398
>>67397
Есть. Даже лучше, он есть и в архиве раздела (следовательно застрахован от проёба пикч, как бывает на архиваче):
https://2ch.hk/spc/arch/2017-12-26/res/242910.html (М)
Просто жалко что он вот так глупо умер.
35 367403
>>67033 (OP)
Существуют ли и возможны ли газовые гиганты, состоящие в значительной мере из кислорода?
36 367404
>>67403
Нит. Кислород имеет свойство - внезапно! - окислять, чем он не преминет воспользоваться и будет связан в молекулах всяких СО2 и иже с ними.
Свободный кислород в атмосфере земляшки результат биологической деятельности.
37 367414
Зависимость от электричества.
Собственно, почему когда человечество изобрело получение энергии от элестричества - перестали предприниматься попытки найти источник энергии лучше? Ведь всё (кроме двс, где сгорание топлива превращается в тягу) построено на электричестве. А оно не идеально.
38 367415
>>67414
Электричество - часть одного из фундаментальных взаимодействий.
Мы и остальные фундаментальные взаимодействия обуздали чтобы перевести их в это.
Если у тебя есть идеи получше - предлагай.
39 367417
В вакууме же постоянно появляются пары частица-античастица, аннигилируют, говорят, что большая энергия у вакуума.
А это значит, что космическое пространство должно что-то весить, нет?
40 367418
>>67417
*обладать некоей массой, простите
41 367419
>>67418

>некоей


Это ты здесь про лифты верши свои выписываешь?
42 367420
>>67419
*некой, само собой. Иногда архаизмы проскакивают, простите.
Нет, это не я.
43 367422
>>67033 (OP)

>упасть в пузырь альбукерке


Ебанный стыд...
Во-первых, Алькубьерре.
Во-вторых, не упасть, а создавать вокруг корабля изнутри (иначе кина не будет).
В-третьих, НАСА искривляет пространство на десятимиллионную часть, контролируя это сверхточными интерферометрами, до самого варп-привода здесь - как до Антарктиды раком.
44 367423
>>67422
Это что, золотой пост?
14619783561920.jpg31 Кб, 500x500
45 367424
>>67423
Урановый
46 367425
>>67423
Иридиевый же
47 367497
Подскажите за дефицитные места для космических аппаратов на ГСО орбите >>367492

Тоже такое видел несколько раз, но с дивана в причины как-то не дошел. Места, в смысле пространства, там с горкой и даже больше, так в чем же дефицит заключается? Спутники мешают работе друг друга? Тяжело с Земли тарелки наводить если кучно летают? Что-то другое?

Насколько сейчас реален дефицит мест для спутников? Сколько может понадобится времени, чтобы аппарат под вохздействием внешних сил сошел с точки стояния достаточно далеко, чтобы можно было занять его место следующему?
48 367499
>>67127

>№82


Да вы даже ещё не начинали
49 367512
Почему нельзя допустим вывести экстремофилов живущих в CO2 среде и отправить их на Венеру для того, чтобы изменить газовый состав планеты?
50 367513
>>67512
Можно. Но пока не вывели надо там полетать и поискать не завелись ли там они сами.
51 367520
>>67512
Это займет так много времени, что пару лишних десятилетий или столетий можно хуи попинать и подумать, на кой ляд оно нужно
52 367525
>>67520
Популяция устойчивых экстремофилов одного вида будет расти в геометрической прогрессии
53 367526
>>67525
И че потом с ними делать?
54 367530
>>67526
писать, какоть
Сами сдохнут, когда весь CO2 преобразится в другие вещества. Кто-то эволюционирует правда
1457313507449.png28 Кб, 186x208
55 367635
Есть сценарий смерти Вселенной из-за большой разрыва, но у меня есть вопрос. Что будет с черными дырами, если Вселенную начнет разрывать? Типа сила расширения Вселенной разорвет их тоже? Как?
Что вообще за хуйня с разрывом Вселенной и что это за сила темной энергии, если она может разорвать все виды взаимодействия? Какая-то хуйня типа массы может искажать метрику пространства, а какая-то хуйня может её расширять допустим. Но должна же быть частица, или бозон, который эту хуйню творит, разве нет? Зачем нам преподают эту противоречивую хуйню?
56 367637
>>67417

>должно что-то весить, нет?


кому должно? Тебе?

>Весить.


Значение знаешь?
57 367638
>>67512

>Почему нельзя допустим вывести экстремофилов живущих в CO2


Можно. Можно даже завезти экстремофилов, которым и СО2 не нужен. Униерсальный растворитель занеси только. А то с Н2О на Венере трудности обнаружились.
Данке за плитиновый вопрос
58 367639
>>67635

>их тоже? Как?


Когда выкатят теорию квантовой гравитации, сразу скажут как. Во всех интимных подробностях. А пока ждите.

>и что это за сила темной энергии


>или бозон, который эту хуйню творит, разве нет?


Самый интересный вопрос, на который нет ответа. Возможно это свойство самого пространства-времени, а может квашеная капуста.

>Зачем нам преподают эту противоречивую хуйню?


У тебя, или у твоего преподавателя какое-то говно в голове. Противоречивого ничего нет. Есть ускоряющееся расширение Вселенной. Это установленный факт. А от чего и почему- это уже объясняющие этот факт теории.

По идее есть конфайтмент в кварковых связях, и его разбирать при разрыве интересно, но оно не инфа 100%. Все остальное решаемо.
59 367649
>>67639
А какая масса у всей Вселенной?
60 367666
>>67649
Масса вещества 6*10^52 кг. Плюс-минус один-два порядка. С учетом темной материи.
61 367671
>>67666
Ок, мне кажется мы к чему-то идем. Можешь объяснить почему конфайнмент и кварк-глюонная плазма не противоречат друг другу? Что-то я читаю одну статью в википедии и там говорится, что выбить кварки из протонов нельзя, а потом говорится что мы уже получили кварк-глюонную плазму на ускорителях. Что это все значит?
62 367705
>>67637
В глаза ебешься, я пофиксил.
63 367708
>>67635
Короче когда горизонт событий вселенной совпадет с горизонтом событий черной дыры, то сингулярность обнажится в то ничто, в которое разрывается вселенная и эта сингулярность зародит новую вселенную.
Трустори, я не бог.
64 367711
>>67708

>сингулярность


Уже не в тренде, учи матчасть.
65 367729
>>67711
Кто пустил кефироблядь в тред?
66 367750
>>67671
Конфайнмент присущ кваркам и глюонам только в "обычных" условиях, а в некоторых особых условиях его может и не быть.
Физический смысл этих "особых условий" все тот же - при низких температурах (формально при Т = 0К) по мере сжатия тяжелого ядра отдельные его нуклоны начинают "налезать" друг на друга (на языке квантовой механики - перекрываются их волновые функции). В результате кварки и глюоны, принадлежащие при обычных внутриядерных условиях отдельным нуклонам, утрачивают своих "хозяев", раскрепощаются - "свой" и соседний нуклоны становятся для них неразличимыми - и начинают свободно перемещаться внутри всего объема сжатого ядра. Конечно, они по-прежнему подвержены конфайнменту, но размер "тюремной клетки" становится намного больше. А если таким же образом сжимаются N ядер, объем возрастает еще в N раз. При достаточно большом числе ядер он может стать вполне макроскопическим и даже огромным. И внутри всего этого объема кварки и глюоны будут перемещаться как обычные свободные частицы (подобно молекулам газа внутри занимаемого им объема). Свойство конфайнмента не то чтобы утрачивается - оно просто становится бессодержательным, происходит деконфайнмент кварков и глюнов. Такое состояние вещества называют кварк-глюонной плазмой.
67 367753
>>67708
Какие-то охуительные истории. Где почитать про это? Или ты из головы это родил?
68 367754
>>67729
Строго говоря, объяснять начальное состояние давно пытаются без сингулярностей. Определенные успехи в построении таких теорий есть, можешь погуглить. Кефир тут не при чем.
69 367810
>>67750
Типа объем к-г плазмы становится большим короткоживущим адроном? ок, спасибо
70 368004
Есть ли какая-то гипотеза отвергающая существование газообразного состояния вещества?
71 368011
>>68004
Да, но её приверженцы имеют не докторскую степень, а запись в медкарте.
72 368012
>>67810
Ну если совсем грубо, то можно и так сказать.
>>68004
Как можно отвергать то, что любой человек может проверить эмпирически, просто сделав вдох, или помахав руками?
73 368014
>>68012
Я не это имел в виду. Я о том что, возможно это и не газообразное состояние, а какое-то иное с совсем иными процессами.

Я почему спрашиваю, в общем, мне сегодня сон приснился: сижу я знач во дворе своего дома, а возле меня какой-то мужик, что-то мне объясняет. Ну, а потом как заорёт "газообразного состояния не существует!". Внезапно вокруг нас начинают появляться раскиданные пл всему двору голубые и оранжевые шары. Внутри голубых галактики, внутри оранжевых хуй пойми что. Так вот, что бы доказать свою правоту, этот мужик взмахивает рукой и голубые шары магическим способом исчезают. Такая вот шиза.
74 368016
Верно ли что дождь выпадает когда количество пара в воздухе больше чем T x 0.003298697 x количество воздуха? (давление постоянное)
75 368017
А оранжевые остаются. Типа, они и без голубых могут существовать.
76 368019
>>68014

>возможно это и не газообразное состояние


Возможно нужно взять определение агрегатного состояния и понять, соответствует ли. Если да, то все ок, если нет, то не ок. А сны иногда просто сны.
77 368021
Может ли существовать полностью водяная планета? Ну или с очемаленьким ядром?
79 368024
>>68014
Газообразное состояние это определение одного из агрегатных состояний.
Есть сущности, состояния описываемые этим термином.
Это все равно, что спорить о существовании света - есть сущность описываемая этим термином и только.
Кто-то может заявлять, что, дескать, нет, на самом деле это колебания эфира, искажения пространства-времени и еще что-нибудь, но термин все равно останется и будет описывать это.
80 368033
>>68024
Это понятно
81 368084
На какое расстояние к Солнечной системе должен приблизиться пульсар/магнетар, чтобы уничтожить обезьянок?
82 368165
>>67497
бамп
83 368181
>>68165
Дефицит реален. Уже сейчас за квоты и места толкаются локтями. Минимально возможно расстояние между спутниками связи не менее 73 км. 1/10 градуса долготы. Определяется шириной луча восходящего сигнала. В особо хитровыебанных случаях, при использовании определенных диапазонов частот и хитрожопых транспондеров, есть возможность набить спутники и сильно плотнее.
Дабы конторы и страны не чистили друг-другу ёбла за место упараши запилен специальный орган ITU, при ООН https://ru.wikipedia.org/wiki/Международный_союз_электросвязи
84 368213
>>67404
Но если газовый гигант будет состоять почти полностью из кислорода, то ему просто нечего будет окислять. Я так понимаю, вопрос был именно о существовании таких гигантов
85 368214
>>68213
А кто отделит кислород от других элементов после большого бадабума сверхновой? Он вместе со всем говном летит и окисляет.
86 368295
Спейсаны, кто может спалить инсайд?
Вот здесь -> https://en.wikipedia.org/wiki/Kepler-1625 <- написано, что 29 октября 2017 г. должен был наблюдаться транзит предполагаемой экзолуны. Знает кто-нибудь о результатах?
Прошло уже порядочно времени, но ссылок на исследования пока нет: http://exoplanet.eu/catalog/kepler-1625_b_i/
87 368296
>>67033 (OP)
Есть ли сегодня способ прослушивания радиочастот в далеком космосе ? Что-то вроде спектрального анализа в мире связи.
88 368305
>>68295

>Знает кто-нибудь о результатах?


Только тот, кто эти наблюдения заказывал. Это дело не быстрое обычно. Либо нихуя не получилось, либо автор статейку пишет.
89 368312
>>68296
Радиоастрономия же, ну.
1b.png7,5 Мб, 3840x2160
90 368314
Мы можем измерить только скорости других галактик относительно нашей, а не относительно друг друга, так ведь? Мне просто интересно - вдруг наше отдаление от других галактик это искусственный процесс, наподобие работы аннигилятора Танева из книги Люди как боги. Там это устройство преобразовывало массу в пространство. Таким же образом некто(3-я по Кардашеву) в нашей галактике мог бы преобразовывать темную материю в пространство, создавая непреодолимый барьер из пространства перед цивилизациями из других галактик.
91 368315
>>68305
Спасибо, анон. Буду мониторить.
92 368317
>>68314
Это все разговоры на уровне фантастического допущения. Всерьез рассматривать такие предположения в рамках научной дискуссии нельзя по нескольким причинам:
1. Такие предположения невозможно проверить и адекватно оценить на данном этапе развития;
2. Такие предположения не решают проблему, а только усложняют, предлагая более сложное объяснение взамен существующего и вводя кучу новых, никак не проверяемых, тезисов и умозрительных понятий. Короче говоря, игнорируются критерий Поппера и принцип Оккама.
3. Идея не имеет приложений.

В общем, серьезно говорить на эту тему станет возможно когда и если будет установлен хотя бы факт существования иных цивилизаций. Про технологию переработки темной материи на пространство я вообще молчу.
1407627480738.jpg45 Кб, 200x200
93 368319
>>68317

>не решают проблему, а только усложняют, предлагая более сложное объяснение взамен существующего и вводя кучу новых, никак не проверяемых, тезисов и умозрительных понятий.


А признать некую темную энергию, которая создает пространство вообще из ничего, естественным образом множа метрику вакуума - это тебе проще, да?
Проверить мы сможем с запуском и введением в строй Веба и прочих сложных инструментов в ближайшие 10 лет. Приложение может быть в том, что мы уже наблюдаем деятельность разума на космологическом масштабе. Вполне возможно, что столкновения галактик это не самый приятный процесс, которого цивилизации 3 уровня стремятся избежать. 3 цивилизации могли бы также и стабилизировать вращение галактик, также создавая и эффект темной материи в гало за счет искусственных черных дыр, хотя для вас это немыслимо с вашей презумпцией естественности.
94 368321
>>68319

>А признать некую темную энергию, которая создает пространство вообще из ничего, естественным образом множа метрику вакуума - это тебе проще, да?


Начнем с того, что темная энергия - гипотеза, это во-первых. Во-вторых, научное сообщество пока не выработало консолидированного мнения касательно ее природы. Так что свои интерпретации о том, что она якобы что-то там создает из ничего, оставь при себе. Если ты считаешь, что твоя модель лучше, аргументируй. Пока что кроме далеко идущих выводов на основе фантазий с твоей стороны ничего не поступило.

Честно говоря, я вообще не понимаю, зачем ты сюда приплетаешь концепцию ТЭ. Если тебе кажется, что эта концепция и то, что ты изложил в своем посте - построения одного уровня и их можно всерьез сопоставлять и рассматривать как альтернативные точки зрения в рамках научной полемики, то ты сильно заблуждаешься.

Хорошо, ты считаешь, что ТЭ - хуита и ее можно земенить твоими фантазиями? Ну рассказывай, что у тебя в итоге получается. Инопланетяне с чудо-агрегатом расталкивают галактики? УХ БЛЯ! Научно просто до охуения!

>Проверить мы сможем с запуском и введением в строй Веба и прочих сложных инструментов в ближайшие 10 лет.


Вот когда введете, тогда и приходите.

>Приложение может быть в том, что мы уже наблюдаем деятельность разума на космологическом масштабе.


Результаты наблюдений в студию. Желательно за авторством компетентных людей, публикующихся в реферируемых источниках.
95 368324
>>68321
Я серьезно думаю, что мои построения одного уровня с гипотезой о темной энергии. Моя гипотеза не предполагает фундаментальных допущений о том, что больше 75% нашей Вселенной заполнено каким-то неизвестным веществом. Это именно гипотезу о темной энергии нужно отрезать бритвой Оккама и поднимать на смех, так как эта гипотеза пытается объяснить наблюдения радикальным образом изменяя соотношение вещества в нашей Вселенной и при этом эта гипотеза никак не состыковывается с Большим взрывом. Конечно я допускаю, что можно преобразовать материю в пространство, и это никак не доказано(точно также как и не доказано, что можно расширять пространство другим волшебным образом), но я склонен допустить, что пришельцам известны и не такие трюки. Если гипотеза темной энергии допускает, что ускоренное расширение начинается само по себе на больших расстояниях, то я заявляю, что оно происходит по чьей-то воле. Точно также как на коллайдерах сталкивая атомы золота можно создать магнитное поле монструозной мощности, которое сильнее, чем таковое в квазарах, я думаю, что разум может создавать уникальные условия для раскрытия законов этой Вселенной, которые не повторяются естественным образом.
96 368326
>>68324
Ты несёшь пиздецовую хуйню. Начнём с того что ты путаешь тёмное вещество которое точное есть и которое известно как физический наблюдаемый факт с тридцатых годов и тёмную энергию.
97 368327
>>68326
начали, сказал что заполнено на 75% веществом, а не энергией, а дальше что?
98 368338
Насколько бы изменились условия существования жизни на Земле, будь у неё больше лун?
99 368339
>>68338
Смотря насколько массивных.
Если бы был Минмус - никак не изменились бы условия, просто поменялись бы мифологии/религии.
Верили бы мы в 9 богов, поклонялись Магнусу, считали время по Массеру и Секунде и кастовали фаерболы не дожидаясь 30 лет сохраняя девственность.
100 368340
>>68339
Понятно. То есть просто предсказать не получится.
А если ещё учесть и их орбитальные характеристики, то предположений ставится бесчисленное множество.
Неужели никаких моделей нет?
101 368341
>>68340
Вообще очевидно же, что лунная система в любом случае стабилизировалась бы на определенной пропорции - либо мелкоспутники, либо относительно крупный, который остальные разогнал бы. Земля не такая большая, как Юпитер, чтобы пасти плеяду крупных тел рядом.
102 368344
>>67423
Нейтрониевый
103 368348
>>68327
Дальше то, что ты путаешь два явления и два абсолютно разных понятия.
104 368363
>>67033 (OP)
Что такое нихуя перед большим взрывом? Почему законы физики именно законы физики и сгенерировались именно такими какими есть? Почему что то сумело в существование и обрело самосознание, а не тупо выполняет алгоритмовую работу и т.п.
105 368364
>>68363

>Что такое нихуя перед большим взрывом?


Хуй знает, это никак не узнать.

>Почему законы физики именно законы физики и сгенерировались именно такими какими есть?


Хуй знает, это никак не узнать.

>Почему что то сумело в существование и обрело самосознание, а не тупо выполняет алгоритмовую работу и т.п.


Вообще-то выполняет, самосознание не отменяет детерминизм.
106 368366
>>68363

>Почему законы физики именно законы физики и сгенерировались именно такими какими есть?


Потому что иначе некому было бы задавать тупые вопросы в спейсаче
107 368367
Кто-нибудь работал/работает в АО ИСС города Железногорск? Интересует как с зарплатой и условиями труда на этом заводе. Планирую туда попасть как закончу ВУЗик.
108 368388
>>68324
Но почему ты считаешь, что тёмные материи-энергии это херня? У нас всего четыре взаимодействия, так? Вполне логично, что может существовать частица, которая взаимодействует только гравитационным способом и не взаимодействует другими. А может и не существовать. Но может ведь и существовать, ей никакие теории не запрещают это делать грубо говоря.
>>68363

>Почему законы физики именно законы физики и сгенерировались именно такими какими есть?


Существует куча теорий. Ну вот тебе навскидку: была целая куча вселенных с чуть-чуть разными законами, и мы ни в одной из них не могли бы развиться ибо законы физики были бы неподходящими, а в этой смогли, поэтому мы именно в такой вселенной. Есть ещё мнение, что когда М-теорию разовьют в достаточной степени и смогут заглянуть чуть глубже, всё встанет на свои места, получится так, что законы и не могли быть другими и нынешнее их состояние — для них естественно.
IMG0714NDEskirtsm.jpg188 Кб, 900x675
109 368391
Кто-нибудь знает, что сейчас с Composite Cryotank Technology? Муск испытывает композитные баки, Орбитал делает композитные бочки для NGL, а чо NASA? Потестировали и хватит, или будут где-нибудь применять?
110 368408
>>68391

>а чо NASA


ICPS композитная, уже намотали и выкатили.

Для первой ступени баки хуярят по-прежнему металлические.
111 368414
>>68408
Ну пиздец, веду SLS-тред, а что ICPS композитная - не знал. Гоните меня, насмехайтесь надо мной.
112 368434
>>68408
>>68414
Да ну, нужны пруфы, иначе анон пиздит.
113 368438
Вероятность самопроизвольного возникновения клетки, примерно равна вероятности самосборки Боинга из кучи металлолома на свалке

Слыхали такое ?
114 368439
>>68438
Хуйня на постном масле. Несравнимые вещи.
115 368441
>>68438
А зачем клетке самопроизвольно возникать? Самопроизвольно возникает какая-нибудь простейшая херня с двумя функциями на всё про всё. Вероятность которой на порядки выше. И та уже эволюционирует в клетку.
инфузория туфелька.webm18,1 Мб, webm,
480x360, 8:00
116 368448
>>68441

>какая-нибудь простейшая херня с двумя функциями

117 368449
>>68448
Клёво, инфузорию сравнить с РНК-организмами. Ты может ещё и разницы между сосновым поленом и баллистической ракетой не видишь?
118 368450
>>68449
я у мамы дипломированный биолог, гугл по слову простейшие выдает это, претензии не принимаю
119 368452
>>68450

>дипломированный биолог


С каких пор в церковно-приходских выдают дипломы?
В любом случае, раз ты не слышал даже о бесклеточных формах жизни, которые прямо сейчас кишат в твоём организме, то я насильно проталкиваю свои претензии тебе в сраку, пока ты стонешь от удовольствия, получая новые знания.
120 368453
>>68450

>претензии не принимаю


Ты, в общем-то нахуй можешь пройти. И дело с концом. Для начала научись разделять эукариот, бактерии, и архей. Потом приходи обратно. Твоя безграмотность и манямирок тут никого не интересует.
121 368455
Посоны, объясните ньюфагу как связана причинность и скорость света? Сколькл раз читал про это, никак не могу догнать, почему причинность будет нарушена если что либо двигается быстрее с?
122 368456
>>68455
https://en.wikibooks.org/wiki/Special_Relativity/Faster_than_light_signals,_causality_and_Special_Relativity
Вот самое простое объяснение.
Я читал статью гораздо сложнее и понял с третьего раза.
TLDR при сверхсвете из-за разницы метрик статичного и движущегося наблюдателя возможна коммуникация назад во времени.
123 368488
>>67033 (OP)
Почему прикрыли Шаттлы, но при этом дрочат на возвращаемые ступени?
124 368492
>>68488

>Почему прикрыли Шаттлы,


Дороха

> при этом дрочат на возвращаемые ступени


Дешевше шатолов
125 368503
>>68488
Шаттлы решили прикрыть после катастрофы Колумбии (хотя они продолжали летать, пока строилась МКС).
Собственно причин было несколько:
Во первых для пилотируемого аппарата он был крайне опасен для экипажа. Для обоих случаев произошедших катастроф, просто не было способа спасения экипажа. И, при этом, о возможных авариях было известно, летали с мыслью "авось пронесет". Например, прогары боковушек были обнаружены ЕМНИП на третьем полете. Пока Челленджер из-за этого не рванул, делали вид что пронесет. Для сравнения, старый союз переживает даже взрыв ракеты под собой (да, было такое) а обе катастрофы пришлись на первые годы эксплуатации а сейчас любой отказ одной системы экипажу не страшен.
Во вторых, оказалось немного дороже, чем рассчитывали. Стоимость межполетного обслуживания и поддержания инфраструктуры съела всю выгоду от многоразовости. Ну и маленькая проблемка с полезной нагрузкой. Шаттл был сверхтяжелой ракетой, выводившей более 90 тонн на орбиту. И запуск стоил как запуск сверхтяжелой ракеты, способной вывести 90 тонн. Но при этом 68,5 из них составлял сам орбитер, а на полезную нагрузку приходилось менее 25 тонн. Ну как если бы сделали ракету, с цеником как у SLS а грузоподъемностью как у протона.
На цену еще повлияли некоторые решения, которые были не особо нужны, но которые сильно переусложнили схему. Например возможность планировать на ~3000 км при сходе с орбиты, добавленная по требованию военных, для операции, которая так ни разу и не проводилась (возможность сделать одновитковый полет на полярную орбиту).

А с возвращаемыми ступенями проще - меньше противоречивых требований, меньше нагрузки (не с орбиты же сходим), меньше требования к надежности (не людей же спускаем) и т.д. В итоге как этот >>68492 сказал, шаттлы дорого, возвращаемые ступени не дорого.
C8aJn-bWAAABCvO.jpg210 Кб, 1094x1094
126 368507
>>68503
Не проще одной картинкой, без бла-бла-бла?
127 368511
Каков диаметр червоточины? Или он изменяется по мере продвижения к нему?
128 368532
>>68507

>Не проще одной картинкой


Нет, не проще. Решение об отмене было принято после катастрофы Колумбии, и связано было с тем, что это был летающий гроб, для которого серьезная авария на самых сложных участках полета (взлет до момента отделения боковушек и сход с орбиты) гарантировано убивала экипаж. И не только "в теории могла убить", а в реальных полетах.
Что касается цены "в лоб" считать не всегда корректно. Это не только ракета для вывода полезной нагрузки, а пилотируемый корабль. И с тем же Союзом (РН) его сравнивать нельзя, нужно сравнивать со связкой Союз(РН)+Союз(КК). Например, NASA платит сейчас 70 лямов за кресло в Союзе. Отправить 8 человек (максимум, который возил Шаттл) обойдется в 560kk, что дороже полета Шаттла.
129 368554
>>68532

>NASA платит сейчас 70 лямов за кресло в Союзе. Отправить 8 человек (максимум, который возил Шаттл) обойдется в 560kk, что дороже полета Шаттла.


Ты тоже ебантяй. Наса платит не за кресло в Союзе, а за полный комплекс услуг, включая тренировки и предполетную подготовку, родной.
wx1080.jpg107 Кб, 1080x673
130 368561
Как на нас отразится создание теории всего?
131 368568
>>68561
Клешни отрастут
132 368574
>>68507
Ебать, 2к за килограмм? Лет десять был уверен что на всех ракетах около 20к/кг
133 368577
>>68507
Сразу видно, что картинку рисовал маскобой
sage 134 368578
>>68577
Я бы даже написал маскосектант.
135 368582
>>68577
>>68578
Он же спейсиксу и контракты нарисовал
136 368587
>>68582

>Он же спейсиксу и контракты нарисовал


И посадки на баржу! И еще УСТАЛОСТЬ МЕТАЛА
137 368589
>>68561
Поймём более менее подробно: на что можно расчитывать, а на что нельзя и т.п.

Ох чувствую с 97.5% вероятностью так и придётся маня компенсировать в космооперах и фентези за путешествия между реальностями, главенство разума над материей, становление своей воли выше законов физики, абсолютная власть над всем вообразимым и невообразимым, всёмогущество и прочее \b\++ и с 70% вероятностью в космо симуляторах за полёты на сверхсвете, прыжки между галактиками, галактические империи, полёты в гиперпространстве, вечную жизнь цивилизации вечная жизнь индивидуума это наверное уже точно плазменнообразная космоопера или даже фентези онли и т.п.

Блять да тут даже сохраняется вероятность этак в 25% что мы так и не сможем в корабли поколений, тотальную колонизацию хотя бы родной галактики в течении десятков миллиардов лет превозмоганий на пердячей тяге, создание полноценного сверхразумного ИИ, наномашины с саморепликацией из говна и палок хотя бы дно типа.

С вероятностью Блять это же просто призматический взгляд моего мозга на будущее через современные обстоятельства и мою интерпретацию накопленного человечеством знаний, по крайней мере то как я могу осмыслить эти знания, ладно насрать я что то сомневаюсь что тут все ожидают экспансии других галактик через миллионы лет, не смотря на всю всратость моей оценки. 5-10% мы не сумеем даже выбраться из своей солнечной системы, нормально её заселив и разослать АМС со спорокораблями с микробами экстремофилами в другие звёздные системы, что бы отсканировать их и простимулировать возникновение жизни там.

И с вероятностью 1-2% мы вообще не выберемся с этой планеты, через жопу организовав исследовательские базы на ближайших телах и нахуярив ещё больше городов на Земляшке, но это уже совсем надо деграёбством со стагнацией и рецессией обмазаться в говнище просто, ибо законы физики ну никак не запрещают начать копать другие около Солнечные небесные тела и замутить там производство. Ебанециумные материалы и технологии тут конечно желательны, но вовсе не обязательны. Соизволив потратить на это несколько столетий\тысячелетий наработы и сотни триллионов бабла, сделать это таки возможно. Ну естественно я не могу исключать возможности того, что всё человечество забьёт на это лысого на всепланетарном уровне и будет аутировать в виртуалках до самого красного гиганта.

Ну или нас может узбагоить мимо джет, йоба вспышка на солнце, планетарный катаклизм различного типа и прочие вселенские говна, вплоть до физических проблем вроде фазовых распадов вакуума и прочего.

Я кончил, можете обоссывать.
137 368589
>>68561
Поймём более менее подробно: на что можно расчитывать, а на что нельзя и т.п.

Ох чувствую с 97.5% вероятностью так и придётся маня компенсировать в космооперах и фентези за путешествия между реальностями, главенство разума над материей, становление своей воли выше законов физики, абсолютная власть над всем вообразимым и невообразимым, всёмогущество и прочее \b\++ и с 70% вероятностью в космо симуляторах за полёты на сверхсвете, прыжки между галактиками, галактические империи, полёты в гиперпространстве, вечную жизнь цивилизации вечная жизнь индивидуума это наверное уже точно плазменнообразная космоопера или даже фентези онли и т.п.

Блять да тут даже сохраняется вероятность этак в 25% что мы так и не сможем в корабли поколений, тотальную колонизацию хотя бы родной галактики в течении десятков миллиардов лет превозмоганий на пердячей тяге, создание полноценного сверхразумного ИИ, наномашины с саморепликацией из говна и палок хотя бы дно типа.

С вероятностью Блять это же просто призматический взгляд моего мозга на будущее через современные обстоятельства и мою интерпретацию накопленного человечеством знаний, по крайней мере то как я могу осмыслить эти знания, ладно насрать я что то сомневаюсь что тут все ожидают экспансии других галактик через миллионы лет, не смотря на всю всратость моей оценки. 5-10% мы не сумеем даже выбраться из своей солнечной системы, нормально её заселив и разослать АМС со спорокораблями с микробами экстремофилами в другие звёздные системы, что бы отсканировать их и простимулировать возникновение жизни там.

И с вероятностью 1-2% мы вообще не выберемся с этой планеты, через жопу организовав исследовательские базы на ближайших телах и нахуярив ещё больше городов на Земляшке, но это уже совсем надо деграёбством со стагнацией и рецессией обмазаться в говнище просто, ибо законы физики ну никак не запрещают начать копать другие около Солнечные небесные тела и замутить там производство. Ебанециумные материалы и технологии тут конечно желательны, но вовсе не обязательны. Соизволив потратить на это несколько столетий\тысячелетий наработы и сотни триллионов бабла, сделать это таки возможно. Ну естественно я не могу исключать возможности того, что всё человечество забьёт на это лысого на всепланетарном уровне и будет аутировать в виртуалках до самого красного гиганта.

Ну или нас может узбагоить мимо джет, йоба вспышка на солнце, планетарный катаклизм различного типа и прочие вселенские говна, вплоть до физических проблем вроде фазовых распадов вакуума и прочего.

Я кончил, можете обоссывать.
138 368590
>>68589

>аутировать в виртуалках до самого красного гиганта.


Таки если мы даже отошлём АМС со спорокораблями к другим системам и заразим тамошние планеты с жизнью, но не сможем в корабли поколений или оцифроваться то Аутировать в виртуалках до самого красного гиганта Правда тут уже скорее до белого, а может и черного карлика если сумеем всё равно придётся только в Солнечной.
139 368592
>>68589
Ну ты намяфантазировал.
Почему в твоём воображении открытие теории всего навсегда запрещает сверхсвет?
140 368595
>>68592
Или наоборот откроет к нему путь, я же сказал

>Поймём более менее подробно: на что можно расчитывать, а на что нельзя и т.п.


>На что можно


Но по современным ожиданиям, в том числе и олдфагов этого треда, должно наоборот по запрещать..
141 368597
>>68592
>>68595
К примеру для самого пространства - скорость света не является пределам, вселенная спокойно может расталкивать две координаты друг от друга на скорости в овер дофига раз больше световой, черные дыры спокойно зажимают в себе материю и энергию невозможным к покиданию своих недр вообще ничего, кроме как тратя массу на гравитационные волны и излучения Хокинга, есть некоторая надежда на то что эти пляски с пространством можно использовать что бы перемещать материю и энергию быстрее света, изолируя её пространственным пузырём или например через червоточину. Может быть ещё можно измудохаться с квантовой парашей в виде туннелирования и запутанности что бы что то перенести Может даже сможем проломиться в другие вселенные... Сможем... или поймём что это анально сложно, требует запредельных энергий, экзотической пораши вроде управляемой тёмной энергией и ебачайших вычислений контроля процесса который не потянет не один компьютер во вселенной и будем дальше >сосать бибу.
142 368598
>>68590
Можно отправить эти споры туда где время течёт очень быстро.
petroswh.jpg118 Кб, 450x538
143 368602
>>68598

>Можно отправить эти споры туда


А еще лучше отправить туда этих спорщиков.
144 368618
Проект освещения территории с развернутой на орбите отражающей поверхности.
Имеет ли он вообще практический смысл? Рассматривается ли его реализация?
145 368621
>>68618
Не имеет, оче быстро над полем пролетает.
146 368622
>>68621
Но можно построить YOBA поверхность и засунуть куда нить на геостационарку, но кому это надо вообще реально есть практический смысл?
147 368623
>>68622
Интересно реально ли таким образом создать источник света визуально ну если не ярче то больше Солнечного? Вызовет ли это эффект большей освещённости некой местности или может даже больший её нагрев чем при свете дня? Можно так попробовать климат подредактировать более пригодными некоторые территории сделать.
148 368626
Что со мной будет происходить, если я прыгну в Юпитер?

Все ли газовые гиганты состоят из всяких водородов и прочих низших углеводородов типа метанов? Возможны ли газовые планеты из каких-нибудь кислородов, аргонов, радонов, неба и аллаха других газов?
149 368627
Говорят, что звук ракетных двигателей роскосмоса у швятого маска из них классическая музыка играет убивает человека. А в чём там поражающий эффект состоит? Если хорошие наушники надеть, то норм будет?
150 368629
>>68627
Нет
Разрешите вас заебать тупыми вопросами? 151 368640
1. я подсчитал и пнул зонд к другой звездюле. И вот пролетев 20-40 АЕ его корпус охладился до значительной температуры и вот удивительно, металлический материал корпуса стал сверхпроводником. Как это отразится на движении аппарата в будущем летя в магнитном поле солнечной системы и галактики? Возможно, что железные или железо каменные метеориты далеко от звздей становятся сверхпроводниками?

2. Вкачаем в Юпитер ещё 7 таких, И вот в его центре начинается процесс термоядерных реакций. Вопрос - останется ли жидкая фаза водорода в глубинах под высоким давлением?
152 368658
>>68450

>гугл по слову простейшие


А я и не просил тебя гуглить "простейшие". Я применил слово "простейшая" как прилагательное, а не как собственное. И по контексту это было ясно видно.
153 368659
>>68623
Больший нагрев до определенного предела. Выше, чем температуру поверхности источника света ты не получишь.
А вообще - возможно в теории, есть маняпланы солетт для подогрева Марса и охлаждения Венеры.
154 368660
>>68626

>Что со мной будет происходить, если я прыгну в Юпитер?


Ты умрёшь.

>Все ли газовые гиганты состоят из всяких водородов и прочих низших углеводородов типа метанов? Возможны ли газовые планеты из каких-нибудь кислородов, аргонов, радонов, неба и аллаха других газов?


Там все есть, газогиганты просто соберают все говно которое видят, и так уж выходит, что большая часть говна вокруг это водород. Остальное по остаточному принципу.
155 368661
>>68627
Там взрывная волна распидорашивает органы. Поди постой рядом с взрывающимся фугасом.
156 368690
>>68661
У меня есть более доступный эксперимент. Можно пойти на концерт, лучше всего какой-нибудь сотона-митол inb4 анон хикка и не пойдёт на концерт, и стать поближе к колонкам, пока не почувствуешь, как внтури тебя вибрируют рёбра. А работающий ЖРД - это в разы ГРОМЧЕЕЕЕЕЕЕ.
157 368693
>>68690
Ну, я как-то минут пять поспал под колонкой в клубе - уставший был и больше суток без сна.
Мне не понравилось, лол.
Но работающий ЖРД выделяет столько энергии, что сравним со взрывом. В зависимости от дальности может вызвать тиннитус, постоянное повреждение слуха, контузию, кому и смерть.
158 368705
>>68660

>ты умрешь


От чего?
159 368714
>>68705
Никогда не слышал о юпитерианских гейшах-ниндзя-убийцах?
Ну как минимум от ебического давления.
Если ты сейчас станешь 358-м хреном, который пискнет про "ну придставьти что я в неуничтожимом кастюми", я приду к тебе домой и запущу тебя в Юпитер
14951262911240.jpg23 Кб, 420x262
160 368715
>>68705
От Юпитера.
161 368716
>>68714
Там же ещё радиоактивный пояс. Или какого-нибудь предел Роша который размозжит его ещё на подлете.
162 368717
>>68716
Предел Роша его не разможит, если он не весит как сраная Луна.
163 368721
>>68714
В обычном скафандре жи. Сколько там в нем давление внутреннее, половина атмосферного? И когда мне пиздарики настанут?

А если я в ниучтажимам кастюми, то до куда я там буду падать? В какой-то момент плотность тамошнишних говн должна стать больше, чем у моей туши, и должен перестать "падать" же? Когда этот момент настанет?
164 368746
Вечерок в хату.

Господа, у меня такой вопрос: какой скафандр использовался для ВКД в советской космической программе с 1971 по 1977 годы, т.е. до ввода в эксплуатацию первого поколения "Орлана"? Или в это время на "Салютах" ВКД вообще не было?
Sharman.jpg102 Кб, 445x640
165 368748
>>68746

>Вечерок в хату.


Жизнь конструкторАм.

>Или в это время на "Салютах" ВКД вообще не было?


Да ето так. Если быть точным, советский гражданин не выходил пробздеться в открытый космос с 1969 по 1977 годы. Если нужны подробности - расскажу.
пик слаборилейтед
image.png1,5 Мб, 700x1074
166 368749
>>68748
Моя бывшая. Выходил с ней в открытый космос.
Какая-то она невзрачная, но что-то в ней есть.
167 368753
>>68749
На первой пикче она мне показалась очень няшной, а потом я погуглил и что-то как-то ну такое.
168 368756
>>68753
Пока что титул самого няшного астронавта у Саманты. Когда услышал её русский так вовсе влюбился.
хитрый грузовичок спиздил твой интерстейдж, пидор.jpg283 Кб, 1000x1494
169 368757
>>68756
Подогнал бы свой эректор к её стартовому приямку? Закатил бы свой бустер в её ангар вертикальной сборки? Запустил бы своего Купера в её Гаргантюа? Вошёл бы своим штырём в её приёмный конус? Закатил бы свою ракету на её стартовый стол? Пульнул бы своим джетом в её межзвёздное пространство? Вошёл бы своим орбитером в её плотные слои атмосферы? Установил бы её на свой Канадарм? Завинтил бы ей пару пироболтов? Инкапсулировал бы свой спутник под её обтекатель? Посадил бы свой Фэлкон на её баржу? Устроил бы ей сброс избыточного давления? Залил бы свой керосин в её баки? Посадил бы свою капсулу в её океан? Устроил бы рандеву своего Союза с её станцией? Уронил бы вещество своего аккреционного диска на её горизонт событий? Загрузил бы свою ракету в её шахтно-пусковое устройство? Запустил бы своего белого карлика в её полость Роша? Провёл бы сближение с её стыковочным узлом? Вставил бы свой тепловой источник в её РИТЭГ? Пробурил бы её грунт своим Drill Bit? Устроил бы ей пассивацию ступени? Направил бы своё рентгеновское излучение на её матрицу? Отстрелил бы свою ступень в её район падения? Устроил бы ей продув трубопроводов? Отметил бы своего субкарлика на её диаграмме Герцшпрунга-Рассела? Устроил бы ей приливный захват своим Юпитером? Установил бы свой носитель на её опорные фермы? Провёл бы её горизонтальные испытания? Покрутил бы своими лопатками в её турбонасосе? Вывел бы её двигатели на режим? Повысил бы давление в её камере сгорания? Заставил бы перегреваться её сопло? Испытал бы свой скафандр в её вакуумной камере? Отполировал бы зеркала в её телескопе? Запустил бы свой факел в её газоотводный канал? Наполнил бы купола её парашютов? Обнаружил бы её коричневого карлика своим широкопольным искателем? Устроил бы прочностные испытания её межбакового отсека? Прошёл бы своим носителем через её точку максимального аэродинамического сопротивления? Поместил бы своего газового гиганта в её обитаемую зону? Запустил бы свой телескоп в её точку Лагранжа? Распылил бы компоненты топлива своей форсункой в её камере сгорания? Установил бы свою башню в качестве элемента её САС? Выродил бы её в нейтронную звезду своим пределом Чандрасекара? Провёл бы огневые испытания на её стенде? Опросил бы своим главным компьютером все её датчики? Проложил бы свою кабельную сеть в её кабель-каналах? Устроил бы космическую гонку между её сверхдержавами? Отправил бы зонд на её Венеру? Исказил бы её магнитное поле своим солнечным ветром? Совершил бы укладку своей полезной нагрузки в её грузовой отсек? Пробил бы своим микроазазаметеороидом корпус её модуля? Подвёл бы свою башню обслуживания к её разьёмам? Загнал бы свой орбитер в её OPF? Надул бы свою шлюзовую камеру в её околоземном пространстве? Залил бы свой хладагент в её систему охлаждения? Взорвал бы свою сверхновую в её звёздном скоплении? Пробил бы пиромембрану в её трубопроводе? Закатил бы свой транспортный агрегат в её монтажно-испытательный корпус? Отправил бы ей пару волн своим радиотелескопом? Закрепил бы её Орион на своём адаптере? Залил бы гидразина в её шаробаллоны? Отправил бы свой луноход исследовать её кратер? Запустил бы свою комету на её высокоэллиптическую орбиту? Уронил бы свой астероид на её селену? Провёл бы мультиспектральную сьёмку её туманностей? Поместил бы своего желтого карлика в её протопланетарный диск? Поднял бы свою громоотводную мачту над её стартовыми сооружениями? Пролетел бы своим Кассини между её колец? Скопил бы свой молекулярный водород в её глобуле? Направил бы своего LORRI на её Плутон? Усадил бы её космонавта на свой ранец MMU? Запустил бы свой протуберанец в её корону? Поместил бы свой керн в её колонковую трубу? Использовал бы Opportunity, если бы она проявила Curiosity к твоему Atlantis?
К чему это я... А, мы же уже выяснили что ты самантофаг, не надо ж говорить за всех.
хитрый грузовичок спиздил твой интерстейдж, пидор.jpg283 Кб, 1000x1494
169 368757
>>68756
Подогнал бы свой эректор к её стартовому приямку? Закатил бы свой бустер в её ангар вертикальной сборки? Запустил бы своего Купера в её Гаргантюа? Вошёл бы своим штырём в её приёмный конус? Закатил бы свою ракету на её стартовый стол? Пульнул бы своим джетом в её межзвёздное пространство? Вошёл бы своим орбитером в её плотные слои атмосферы? Установил бы её на свой Канадарм? Завинтил бы ей пару пироболтов? Инкапсулировал бы свой спутник под её обтекатель? Посадил бы свой Фэлкон на её баржу? Устроил бы ей сброс избыточного давления? Залил бы свой керосин в её баки? Посадил бы свою капсулу в её океан? Устроил бы рандеву своего Союза с её станцией? Уронил бы вещество своего аккреционного диска на её горизонт событий? Загрузил бы свою ракету в её шахтно-пусковое устройство? Запустил бы своего белого карлика в её полость Роша? Провёл бы сближение с её стыковочным узлом? Вставил бы свой тепловой источник в её РИТЭГ? Пробурил бы её грунт своим Drill Bit? Устроил бы ей пассивацию ступени? Направил бы своё рентгеновское излучение на её матрицу? Отстрелил бы свою ступень в её район падения? Устроил бы ей продув трубопроводов? Отметил бы своего субкарлика на её диаграмме Герцшпрунга-Рассела? Устроил бы ей приливный захват своим Юпитером? Установил бы свой носитель на её опорные фермы? Провёл бы её горизонтальные испытания? Покрутил бы своими лопатками в её турбонасосе? Вывел бы её двигатели на режим? Повысил бы давление в её камере сгорания? Заставил бы перегреваться её сопло? Испытал бы свой скафандр в её вакуумной камере? Отполировал бы зеркала в её телескопе? Запустил бы свой факел в её газоотводный канал? Наполнил бы купола её парашютов? Обнаружил бы её коричневого карлика своим широкопольным искателем? Устроил бы прочностные испытания её межбакового отсека? Прошёл бы своим носителем через её точку максимального аэродинамического сопротивления? Поместил бы своего газового гиганта в её обитаемую зону? Запустил бы свой телескоп в её точку Лагранжа? Распылил бы компоненты топлива своей форсункой в её камере сгорания? Установил бы свою башню в качестве элемента её САС? Выродил бы её в нейтронную звезду своим пределом Чандрасекара? Провёл бы огневые испытания на её стенде? Опросил бы своим главным компьютером все её датчики? Проложил бы свою кабельную сеть в её кабель-каналах? Устроил бы космическую гонку между её сверхдержавами? Отправил бы зонд на её Венеру? Исказил бы её магнитное поле своим солнечным ветром? Совершил бы укладку своей полезной нагрузки в её грузовой отсек? Пробил бы своим микроазазаметеороидом корпус её модуля? Подвёл бы свою башню обслуживания к её разьёмам? Загнал бы свой орбитер в её OPF? Надул бы свою шлюзовую камеру в её околоземном пространстве? Залил бы свой хладагент в её систему охлаждения? Взорвал бы свою сверхновую в её звёздном скоплении? Пробил бы пиромембрану в её трубопроводе? Закатил бы свой транспортный агрегат в её монтажно-испытательный корпус? Отправил бы ей пару волн своим радиотелескопом? Закрепил бы её Орион на своём адаптере? Залил бы гидразина в её шаробаллоны? Отправил бы свой луноход исследовать её кратер? Запустил бы свою комету на её высокоэллиптическую орбиту? Уронил бы свой астероид на её селену? Провёл бы мультиспектральную сьёмку её туманностей? Поместил бы своего желтого карлика в её протопланетарный диск? Поднял бы свою громоотводную мачту над её стартовыми сооружениями? Пролетел бы своим Кассини между её колец? Скопил бы свой молекулярный водород в её глобуле? Направил бы своего LORRI на её Плутон? Усадил бы её космонавта на свой ранец MMU? Запустил бы свой протуберанец в её корону? Поместил бы свой керн в её колонковую трубу? Использовал бы Opportunity, если бы она проявила Curiosity к твоему Atlantis?
К чему это я... А, мы же уже выяснили что ты самантофаг, не надо ж говорить за всех.
170 368758
>>68748
Излагай, Антуан. Было б здорово, если еще и со ссылками на книжки годные да не лживые.
feel 3.jpg57 Кб, 1280x720
171 368759
>>68757
Это паста откуда-то? Или сам щас сгенерил?

>мы же уже выяснили что ты самантофаг, не надо ж говорить за всех.


То чувство, когда ты настолько олдфаг доска настолько полуживая, что тебя узнают.
2001.06.16ISS-02YuryUsachyovwithfoodintheZvezdaservicemodule.jpg1,4 Мб, 3040x2008
172 368763
>>68758
Короче, после перехода между Союзами 4 и 5 то было не нужно - военные ДОС не об этом, сиди да фотографируй американцев, ну и вообще с первыми станциями было тухловато - много фейлов. Вот начиная с Салют-6 началась настоящая орбитальная житуха - вылетали экспериментировать, вылетали чинить.
Ссылок на подробные книги не найду, но если хочешь в общих чертах ознакомиться - Мировая пилотируемая космонавтика. История. Техника. Люди. (Батурин Ю.М., 2005), найдётся быстро. Книга - 100 из 10.
Есть ещё Салют-7. Записки с "мертвой" станции от Виктора Савиных (он туда летал, ну ты понел), но я её ещё не читал, так что оголтело советовать не имею права.
>>68759

>Это паста откуда-то? Или сам щас сгенерил?


Я её раньше сгенерил, не помню уже зачем.

>доска настолько полуживая, что тебя узнают.


Да ну ты чё, сейчас спейсач намного активнее, чем года три или четыре назад, когда почти вся жизнь была только в огуречном треде + 3 поста за сутки в остальных.
173 368767
>>68763
Спасибо. Про первую не слышал. Сам еще никак не доберусь до главных талмудов - дневники Каманина и мемуары Чертока. И жаль, конечно, Королев ничего не оставил после себя.
174 368768
>>68763
И еще: есть ли какие-нибудь книги, освещающие работу на ДОС "Салют-3" и "Салют-5"? В интернетах нет даже их фото в полете. Или до сих пор секретно?
2016.06.11rnf1KMS.jpg431 Кб, 1600x1200
175 368769
>>68767
Да, Каманин и Черток это обязательная программа, хотя Каманина читать тяжело. А вот Черток заходит только успевай страницы листать, я тебя уверяю. Главное начало первого тома перетерпеть.
А к "Мировой пилотируемой космонавтике" советую даже до них присмотреться - это считай обзор всех пилотируемых полётов (и наших, и американских) до 2003-го года, с охуительными (в хорошем смысле) историями, интересными подробностями и фактотой (только про катастрофу Колумбии там лучше читать перепроверяя в других источниках, помню я там на какие-то неточности наткнулся, когда для фактотреда пасту пилил). На Каманина и Чертока частично опирается кстати. В общем базарю, ещё захочешь.
Salyut-31364725657saljut3.jpg177 Кб, 529x655
176 368770
>>68768
Ты не поверишь, но всё в той же книге >>68769
А с пикчами и правда бида, я помнится нашёл только одну (1) фотографию Салюта-3 на орбите, и то шакальную (пикрилейтед), и столько же фотографий Салюта-1. И всё!
177 368771
>>68770
Спасибо. Наконец-то с кем-то нормальный диалог в /spc/ за все время.
ep5.jpg51 Кб, 630x410
178 368774
>>68717
А если он весит как мамка опа?
179 368775
>>68769

>хотя Каманина читать тяжело.


а что именно тяжело? Дневники читаются легко

мимо-с-дивана
180 368776
Шо думаете о сиём видео?
https://youtu.be/bPyS0AJ4eY4
image.png821 Кб, 779x960
181 368777
>>68776
Я знаю, что не надо судить книжку по обложке, но даже открывать не буду, вот что я думаю.
Image1x.jpg28 Кб, 624x341
182 368778
>>68774
Объекты мамкокласса не укладываются в рамки любых известных сегодня теорий физики, не могу ничего сказать об этом.
>>68775
Ну там типа так:
Карочи у нас в производстве такие-то корабли, космонавт слеатал так и так писол, какол, планируем Союз вот допиливать.
Ты такой - интредастинг. А потом:
Ну сьездил я к внуку, жена болеет вот.
Ты такой - ну ёб твою мать.
Возили Терешкову показывать в дружественные страны, писола, какола, ещё там Луна сфейлилась, Королёв пидор, хотя не пидор, но как бы и пидор
Ты такой - ну есть интересные фактики, хотя уныловато.
Был на партсобрании, полстраницы имён присутствиющих, на партсобрании сидли и пиздели, видел то ебло и это, а потом возили.
Ты такой - БЛЯЯЯЯЯЯЯЯЯ
Эти пидоры срывают сроки, вон те пидоры вставляют палки в колёса программе, десять страниц с описаниями разговоров с мудаками в правительстве, а Королёв всё-таки не пидор, а вот Мишин - совершенно определённо пидор
Ты такой - скучно, но очень познавательно.
Ну в общих чертах как-то так. То есть там есть очень интересные и охуенные вещи, но в целом чтиво тяжёлое.
183 368780
>>68778
как молодому хую поколению мне интересно всё полряд
STS-127369036mainjsc2009e143029hires.jpg1,5 Мб, 3536x2775
184 368781
>>68780
Ну завидую тебе тогда, я уже похоже становлюсь немолодым сморщенным и вялым хуем, хотя и до старого ещё очень далеко.
Но пиздец время летит, ещё казалось недавно на Constellation фапал и живые трансляции пусков шаттла смотрел, а уже 7 лет прошло как ни того, ни другого в живых нет.
185 368787
Как меряют массу супермассивных ЧД?
186 368788
>>68787
Примерно чувствуют.
Я бы сказал, что по орбитам звезд, массы которых посчитать можно благодаря главной последовательности, но потом понял, что так можно посчитать только массу Стрельца А*
А в других галактиках же звезд не видать уже, так что двачую вопрос.
187 368796
>>68721
Бимп, блджад
blob92 Кб, 348x348
188 368806
>>67127
В конце 13-го я помню первые треды начали появлятся. Пять лет назад, твою же ж мать, первые тред по инженеграм, тот самый тред астрофотографий, активный ксп тред, рождение навозного жука. А самое главное, что большинство людей здесь осталось тех же.

>>68796
Во-первых, с какой высоты ты спрыгнешь? Если хотя бы с 5 тысяч километров над атмосферой, то ты скорее всего просто сгоришь, т.к ты достигнешь скорости около 15 км/с, если выше, то ты отхватишь нихуевую долю радиации за те 5-6 минут, пока будешь лететь. И плюс ты сгоришь.

Если прыгнешь на границе атмосферы, то тебе нужно будет падать еще 5 тысяч километров до границы, когда давление сравняется с атмосферным. Какую при это ты разовьешь скорость я не знаю, это надо считать с учетом площади поверхности тебя и градиента плотности атмосферы, но с дивана скажу, что в принципе ты долетишь туда одним куском.

На ~90 км ниже этой границы давление уже 10 бар, т.е каждые ~10 км добавляется одно атмосферное к уже имеющемуся. Плюс растет температура. Плюс радиация. Плюс ветер. Т.е где-то к трети пути тебе станет уже очень плохо и наверно где-то к четверти-половине ты уже умрешь.
189 368818
>>68721

>В обычном скафандре жи


А надо в батискафе!
190 368825
Горизонты в этом году встретятся с объектом икс?
191 368843
>>68825
В январе 2019
192 368845
>>68806

>15км/с


Терминальная велосити, ты где?
193 368847
>>68845
Вне атмосферы?
194 368896
Что мощнее, среднестатистический ионник или пердячая тяга?
195 368898
>>68896
Пердячая тяга.
196 368932
>>67033 (OP)
Почему не рассматривается вариант жизни на других планетах под поверхностью? Ведь это логичнее и даже безопаснее чем жить под открытым небом.
197 368935
>>68932
Рассматривается, см. Н.Носов, 1965
198 368936
>>68932
ХЗ как искать потомушто. Вон из ледяного Энцелада плюмы хуячат - подлетай да анализируй, а если они просто тихонько сычуют в каменной планетке на глубине, то как их обнаружить?
199 368944
>>68935
Годная идея была там описана.
200 368976
>>68936

>а если они просто тихонько сычуют в каменной планетке на глубине, то как их обнаружить?


Импактором или ядерным зарядом расхуячить
201 369029
>>68976

>ядерным зарядом расхуячить


В звездный флот тебя бы не взяли
202 369055
>>68976
Вот я опасаюсь что пришельцы не примут аналогичный метод к поиску жизни на нашей планете.
203 369061
Я конечно понимаю, что скорость света в вакууме неизменна, но мне как человеку который разбирается в астрономии на уровне "гыы, вон ковш, который большая медведица! " не даёт покоя мысль о том, что солнечный свет притягивается планетами, мимо которых пролетает, а значит чем больше звезда, которая излучает свет, тем меньше (изначальная) его скорость. Об этом кто-нибудь думал, или скорость света в вакууме неизменна - это неколебимая аксиома?
image.png133 Кб, 571x380
204 369062
>>69061
В концепции современной теории гравитации, тела не то что бы тянут, в том смысле, как одеяло на себя, а деформируют пространство-время вокруг себя. Так что да, скорость света в вакууме неизменна и постоянна. Вот пространство и время - нихуя.
1514932533175853170.jpg104 Кб, 668x769
205 369080
>>69061

>не даёт покоя мысль


>свет притягивается

206 369106
>>68640
ребят! У меня ракета заправленная стоит, я не могу полететь без ответов на вопросы. Дайте команду!
207 369108
>>69106
Илон, ты запарил переносить, так запускай уже.
208 369173
>>69106
ПОЕХАЛИ
209 369232
Я правильно понял что зарегистрированные гравитационные волны с ЛИГО и еще каких-то нонейм аппаратов уже официально подтвердили существование черных дыр?
210 369233
>>69232
А разве их существование не было доказано раньше и не оспаривается в научной среде?

мимо-с-7-классами-церковно-приходской-школы
211 369234
>>69233
и оспаривается*
212 369235
>>69232
Нет не правильно. То что научное сообщество поддерживает эту теорию в рамках ОТО не значит, что это 100% правильная интерпретация. Мы можем только предполагать что эта теория более вероятно является верной.
213 369237
>>69235
С такой логикой можно с уверенностью сказать, что сейчас подтверждено ноль теорий.
214 369240
>>69237
Почему это? Теории то подтверждены, только они подтверждены необъективными способами и в науках которые мы сами придумали, а не они существовали до нас. Ведь если наша наука ошибочна, но мы об этом не знаем, поскольку объективно это узнать не можем, то и говорить о том что мы знаем объективную истину, а не заблуждаемся, получается не совсем верно. Мы можем говорить что повторили эксперимент или провели наблюдения 10000 раз и результат не изменился. Но это дает нам знание, что мы наблюдали и делали что то 10000 раз, а не объективную истину, которую мы узнать не можем.
215 369247
Хммм... Если ЛИГО и прочие нонейм аппараты опредилили что есть большое кол-во блуждающих ЧД, а так же двойных блуждающих ЧД - интересно что было бы если бы такая штука в 100 солнечных масс прошла сквозь солнечную систему? Даже не близко, а так, в районе пояса Оорта.
216 369250
>>69240
Ну да.
Вообще, начнём с того, что мы на линзирование от чёрных дыр нормально не смотрели. И они немношк слишком далеко от нас. Так что мало что можно сказать наверняка даже в рамках "проверим сто раз". Закончим тем, что хрен знает, что там происходит рядом с гипотетическим горизонтом событий. Слишком экстремальные условия, сложно сказать что-то наверняка даже в рамках устоявшихся теорий, хотя кто-то вон умудряется. Но... насчёт того, что происходит под ГС у нас нет даже предположений. Теория не выработана, её не от чего разрабатывать. Так что вряд ли регистрация слияния как-то сильно повлияла на наши теории. Алсо, что она ещё раз железно доказала, так это то, что выводы из имеющихся теорий, говорящие нам, что два массивных объекта при приближении друг к другу отдают часть массы (энергии) в виде гравитационных волн, оказались правдивы. Это довольно значимо, имхо, ведь такому эффекту подвержены, в сущности, вообще все тела, хоть он и очень мал если мы говорим о чём-то менее значительном, чем слияние ЧД/нейтронных звёзд.

Объясните мне лучше вот что. Почему часть массы двух ЧД не разбегается в ужасе во всех направлениях непосредственно перед слиянием? Вот, предположим, ГС двух ЧД касаются друг друга. В этот момент на точку соприкосновения двух ГС влияет как гравитация одной ЧД, так и гравитация противоположной, таким образом гравитационные силы друг друга обнуляют... В общем, к этому моменту я уже и сам обо всём догадался, но всё равно интересно: если бы масса ЧД была сосредоточена не в одной точке пространства, а немного распределена около центра, то было бы возможно вытащить часть массы притяжением другой чд? Так, блжад! Сейчас мне в голову пришли сразу две вещи. Во-первых, в чд, конечно, много частиц и сила притяжения между ними огромна. Но ведь частицы обладают не только притяжением и сила электромагнитного отталкивания должна быть ещё выше, нет? Разве около самого центра ЧД горизонт событий не должен начать пропадать? И ещё одна вещь, которая идёт к конфликт со всеми предыдущими. Для частиц ведь замедляется время и они не должны упасть за ГС. То есть если ЧД сожрала звезду, то масса звезды должна быть растянута по зоне около ГС, нет? В таком случае при сближении двух ЧД часть массы и подавно должна куда-нибудь свалить, нет?
216 369250
>>69240
Ну да.
Вообще, начнём с того, что мы на линзирование от чёрных дыр нормально не смотрели. И они немношк слишком далеко от нас. Так что мало что можно сказать наверняка даже в рамках "проверим сто раз". Закончим тем, что хрен знает, что там происходит рядом с гипотетическим горизонтом событий. Слишком экстремальные условия, сложно сказать что-то наверняка даже в рамках устоявшихся теорий, хотя кто-то вон умудряется. Но... насчёт того, что происходит под ГС у нас нет даже предположений. Теория не выработана, её не от чего разрабатывать. Так что вряд ли регистрация слияния как-то сильно повлияла на наши теории. Алсо, что она ещё раз железно доказала, так это то, что выводы из имеющихся теорий, говорящие нам, что два массивных объекта при приближении друг к другу отдают часть массы (энергии) в виде гравитационных волн, оказались правдивы. Это довольно значимо, имхо, ведь такому эффекту подвержены, в сущности, вообще все тела, хоть он и очень мал если мы говорим о чём-то менее значительном, чем слияние ЧД/нейтронных звёзд.

Объясните мне лучше вот что. Почему часть массы двух ЧД не разбегается в ужасе во всех направлениях непосредственно перед слиянием? Вот, предположим, ГС двух ЧД касаются друг друга. В этот момент на точку соприкосновения двух ГС влияет как гравитация одной ЧД, так и гравитация противоположной, таким образом гравитационные силы друг друга обнуляют... В общем, к этому моменту я уже и сам обо всём догадался, но всё равно интересно: если бы масса ЧД была сосредоточена не в одной точке пространства, а немного распределена около центра, то было бы возможно вытащить часть массы притяжением другой чд? Так, блжад! Сейчас мне в голову пришли сразу две вещи. Во-первых, в чд, конечно, много частиц и сила притяжения между ними огромна. Но ведь частицы обладают не только притяжением и сила электромагнитного отталкивания должна быть ещё выше, нет? Разве около самого центра ЧД горизонт событий не должен начать пропадать? И ещё одна вещь, которая идёт к конфликт со всеми предыдущими. Для частиц ведь замедляется время и они не должны упасть за ГС. То есть если ЧД сожрала звезду, то масса звезды должна быть растянута по зоне около ГС, нет? В таком случае при сближении двух ЧД часть массы и подавно должна куда-нибудь свалить, нет?
217 369279
Расскажите что-нибудь интересноге, или ссылку дайте...
219 369342
>>69247
Даже если она будет в 1 солнечную массу и пройдёт в 10 раз дальше, чем облако оорта, нашей солнечной системе наступит пиздец.

Играй в Orbiter, чтобы познать что такое космические расстояния, силы и время. В космосах достаточно тихого пука, чтобы всё пошло по пизде.
220 369344
>>69342
А ты не доверяй игорам на все сто
Внешняя граница облака Оорта – 2 световых года. На таком расстоянии уже пролетали другие звёзды, раньше. А пролет на расстоянии в два световых года мимо звёзды массой в солнечную и чд с той же массой – одно и то же.
Сомневаюсь что на таких расстояниях даже звезда/чд в сотку солнечных масс сможет сделать что-нибудь ещё кроме как взбаламутить кометное облако
221 369347
>>69324
Хуле ленивый ушлепок перестал вотыф делать?
222 369350
Можно ли спейс шаттл называть ракетой?
223 369351
>>69344
Сотка масс может чутка колбаснуть орбиты планет и они будут не такими круглыми. Но да, ничего фатального не должно быть.
224 369371
>>69350
Скорее космоплан
225 369377
>>69371
Я видел фразу от ученого изнасиловавшего журналиста "ракета Челленджер взорвавшаяся в 1986"
Это вообще легально?
226 369378
Ну как вы там, астрономы. Нашли уже 9 планету? Есть новости?
227 369379
>>69378
Сто лет назад как. Потом решили её планетой не считать. А что?
228 369380
>>69379

>Потом решили её планетой не считать.


Потому что открыли много похожих на него карликовых планет. Это как со спутниками: они достаточно самобытные, чтобы быть просто спутниками. Просто так вышло, что они привязались к более массивным планетам. Но Плутон и подмётки не годится девятой, если её, конечно, найдут.
229 369381
>>69379
Он про планету X
230 369383
>>69347
НЕ РАЦИОНАЛЬНО, НЕ ПРИНОСИТ ПРИБЫЛИ, ЖРЁТ МНОГО ЭНЕРГИИ. НЕТ ПРАКТИЧЕСКИХ ЗАДАЧ. А ОН ЖЕ УЧЁНЫЙ ОН ЖЕ РАЦИОНАЛЬНО МЫСЛИТ!
231 369390
>>69377
А что, шаттл уже не ракета?
232 369391
>>69237
Это тупой пиздунец-фаллософ, не слушай его.

В рамках научной методологии это вполне себе норм подтверждение. Единственно, оче близко к ГС у ЧД могут быть свойства, отличающиеся от предсказанных в рамках ОТО. Но в доступных нам масштабах, если объекты излучают гравитационные волны как черные дыры, то они, внезапно, и есть черные дыры.
233 369404
Теория струн вообще признается ведущими мировыми учеными?
Или это бредни упоровшихся математиков?
Просто читал одну книгу по этой хуйне, ну слишком много нестыковок, слишком много самих разных теорий струн.
Плюс она не выдает никаких предсказаний, поддающихся экспериментальной поверке сейчас или даже в ближайшем будущем.
Предлагая или шатать измерения на масштабах меньше планковских или построить коллайдер размером с галактику.

И еще вопрос. Че там с суперсимметрией?
234 369408
>>69404

>Че там с суперсимметрией


Ее нет. А значит и теория суперструн, м-теория и иже с ними в говне.
235 369409
>>69235
Ты хуй. ОТО предсказала гравитационные волны и это предсказание подтвердилось. Больше чем это никому от теории не нужно
236 369415
>>69404
Её НИХРЕНАЖ-эффект был в том, что она помирила квантмех и теорию относительности. Когда поняли, что несостыковок что-то многовато, выдумали суперструны. А потом М-Теорию, в которой несостыковок уже вроде сильно меньше, но там начинается такая математика и такие объемные расчёты, что её до сих пор особо не разгребли и сидят обдумывают это всё. То есть её значимость вполне себе видна, читай первое предложение, но при этом она оче сложная.
237 369433
>>69409
При чем тут это? Возможно я выразил слишком размыто свою мысль. Я всеми руками за научный метод, только нужно доверять, но проверять, оставаться немного скептиком. А не то что теория чем то плоха, и что я самый умный.
238 369438
>>69433
Отмазался ОК, но на будущее лучше так не делай, а то примут за растекшегося кефиробога.
239 369469
>>69390
Нет, система Спейс Шаттл не является ракетой.
unnamed.png398 Кб, 1280x900
240 369494
Парни, парни, я только что видел МКС!
Впервые в жизни!
Я просто охуел. Яркость -2.9, я думал ничего не увижу, а она яркая пиздец. Заснял видео, но там всё размыто и хреново видно.
Теперь буду охотиться на все иридиумы и все пролёты МКС.

Это я себе недавно андроид телефон купил, поставил это https://play.google.com/store/apps/details?id=com.runar.issdetector
Кто ещё не наблюдал, попробуйте, зависимость лютая.
Яркая как на видео https://www.youtube.com/watch?v=v0rVCOtypac
241 369502
>>69494
И вообще я думал она будет как вспышка, если не понял куда смотреть, то пропустил.
А она, оказывается, яркая на всём пролёте. Не заметить невозможно.
242 369513
>>69250
Когда частица падает на горизонт она замедляется. Но её масса суммируется с массой частицы, и горизонт расширяется.

И ты ошибаешся что нет теорий, что под трусиками у чёрного щита.
Есть вполне себе нормальное предположение вращающейся намагниченой ЧД что это в прямом смысле дырка в пространстве-времени, в которую утекают вещества.
А ГВ образует только 2 объекта.
243 369517
>>69469
А как на счёт того аргумента, что у системы Спейс Шатл ни больше и ни меньше возможностей и требований, чем у ракеты.
Это сверхтяжёлая не симметричная ракета, у которой функцию теплового щита выполняли крылья. А ещё у неё был дизайн самолёта, что нравилось военным. А вместо парашюта планёр.
244 369854
ХМ, почему говорят что луна бесполезная и делать там нечего? Ведь она одной стороной обращена к земле - можно на ней какую-нибудь йоба отражатель поствить или что-ниьбудь такое?
245 369855
объясните мне, тупице, какова гравитация плутона по ощущению для стоящего на его поверхности человека с точки зрения такой игры как ксп?
как на минмусе? как на гилли? там просто наглядно вижно жэ в движении, так сказать
246 369880
>>69855
Вот примерно симуляция как человек бы прыгнуть мог на разных объектах. Там плутон в выборе есть
http://cosmos-book.github.io/high-jump/index.html
247 369890
>>69880

> всего 7-8 метров


хм
интересно, спасибо
а можно там как=то прибавить вес скафандра космонавта для сверки данных? это же ещё чуть ли не центнер веса при той же силе прыжка, что снижает высоту.
и да, если - чисто вдруг - пролетаешь ты такой с парой друзей на небольшой ракете и вынужденно садишься на плутон, и вдруг понимаешь, что сел немного не туда, можно будет выйти из ракеты и переставить её куда нужно? то есть, конечно да, но я имею в виду, насколько это будет сложно.
248 369892
Невесомость в самолете-лаборатории чем-нибудь отличается от невесомости на орбите?
249 369895
>>69502
>>69494
Перепости в тред линзоебов, там тебе рады будут
HoleInOne-full.jpg172 Кб, 1000x1000
250 369896
>>69890
Это будет рискованно. Объекты весят меньше, но масса же у них есть. Вот на Луне видишь этот камень? Он размером с дом, но он скользил как на льду по поверхности. Так и твоя ракета может упасть на тебя довольно легко и придавить при неверном движении, а может разрушиться что-то внутри неё при падении. Лучше быть жутко аккуратным. То есть нужно опоры для ракеты специально широкие делать, чтобы она не наклонялась.
251 369898
>>69892
конечно, ты падаешь вместе с самолетом вниз, но не испытываешь при этом сопротивления воздуха как при прыжке с парашютом, когда этот воздух тебе в лицо и уши орет. это лишь иллюзия невесомости на очень короткое время, не дольше 1 минуты.
252 369899
>>69898
ты не пояснил

мимо
253 369901
>>69892
Ничем кроме краткости.
254 369928
там такая аксиома: с помощью линз нельзя сфокусировать и получить температуру больше чем температура тела излучаемого эту энергию. То есть в фокусе мы не получим больше 6500 градусов от солнца даже если линза будет с планету целиком. А как с вогнутыми зеркалами в этом случае?
Вы кстате так и не ответили на мои вопрос >>68640 по этому я и проебал сегодня военный спутник.
255 369956
>>69928

> по этому я и проебал сегодня военный спутник


Маск, залогинься.
256 369970
>>69898
На мкс ты тоже падаешь с воздухом внутри нее.
257 369988
Ша там с Emdrive ?
258 369997
>>69988
Улетел вместе с Зумой на альфу центавру
image.png512 Кб, 492x604
259 370273
А космонавты дрочат? А какого это в невесомости?
260 370278
>>70273
Нет. Им по 60 лет и они заняты работой до усрачки. В такой ситуации ты вообще забываешь, что у тебя член есть если рядом не болтается Саманта Кристофоретти, конечно.
261 370283
>>70278
Кристофоретти няша.
262 370284
>>70283
ЕЙ полтос.
263 370285
>>70284
Всё равно бы вдул, почти ровесница же.
264 370288
>>70285

>полтос


>сидеть на анонимном форуме для девочек-анимешниц

265 370289
>>70288
Я этот тред знаешь сколько лет перекатываю уже?
И ей всего 40.
И выглядит в разы няшнее, чем 20+-летние прошмандовки, которых я иной раз на улице наблюдаю.
266 370290
>>70283
Удваиваю.
267 370292
>>70273
Пик реален что ли?
268 370293
>>70292
Вполне возможно.
Ты слышал, есть такая штука уже не один десяток лет, называется "цветной принтер". Любой может купить и напечатать что угодно кроме денег
RVYp80s.jpg215 Кб, 2160x1440
269 370410
Почему США отказались от винрарного Saturn V? Только из-за большой стоимости?
270 370411
>>70410
Нет задач. Союз обосрался в гонке и огромная дурында ничего уже не показала бы.
271 370413
>>70410
Что ты на нём запускать будешь? Где клиентов найдёшь?
272 370430
>>70410
Забавно. "Ангара" есть, а "Протона" и "Энергии" - нет.
Авдтоий 273 370473
Спейсаны, хочу запустить свой спутник на НОО.

Будут ли со мной сотрудничать компании, которые делают комплектующие - гиродины, маневровые движки, электронику и т.п. и т.д.? Не пошлют ли они меня нахуй?

Идея такая - спутник я разрабатываю сам, закупаю все комплектующие у смежников, собираю, запуск отдаю роскосому.

Бабки - не проблема.
274 370475
>>70473
Давай пока бабки мне переводи 150 тысяч рублей. Анонс решим как тебе помочЪ.
275 370476
>>70475
Смета где? На что собираешься тратить?
276 370477
>>70476
На представительские расходы пока..
277 370478
>>70477
Смета где, ещё раз спрашиваю?
278 370481
>>70478
Костюм 30000 рублей, Коньяк 10000 рублей, Проститутка 20000, Аренда апартаментов 30000, На оперативные расходы 60000.
279 370490
>>70473
С вероятностью в 80% на тебя посмотрят как на дурачка, как на Маска в своё время. Ишь ты, выискался, хуй с горы спутник собирать будет. Но продать может и продадут.
280 370492
>>70490
>>70473
А мне кажется вполне реально. Кубсаты сотнями запускают.
281 370495
>>70481
Проститутка, коньяк... Ну признайся, ты просто хочешь красиво и дорого расстаться с девственностью. Причем, судя по тратам на аппартаменты и опретавные где нить во в вьетнаме или тае

>>70490
А вот любопытно, по ПП РФ от 22.02.2012 N 160, ебать мозги станут? Если буду выступать не как ООО или ИП, а как просто физлицо?
Могут ли сюда приплести ПП РФ от 15 апреля 1995 г. N 333?

И что про связь? Нужноли будет получать лицензию? И кто этим занимается для космоса, ну наверняка же не ФГУП «ГРЧЦ»?

>>70492
Мелко мыслишь.
282 370497
>>70430
HUMAN SPACEFLIGHT тебя не смутило?
283 370498
>>70473
Пошлют, потому что твой заказ на 2 "вот эти херовины за пару тысяч" никому не интересен. Хоть платы какие-то, хотя железяка. В розницу таким не торгуют. Это вообщем типовая проблема для любого подобного начинания. А у тебя весь спутник из десятка таких херовин состоит.

Да и что ты собрался делать со спутником после? Продавать свои бип-бип Роскосмосу? Держи карман шире. Если правда интересно, почитай прохладные того же зеленого кота про его даурию
284 370499
>>70497
А энергия действительно где?
>>70492
Двачую этого
285 370502
>>70499
А сколько космонавтов вывели на орбиту на Энергии?
286 370503
>>70502
Ну в теории до 10, на буране
на ангаре тоже не летали и что
287 370504
>>70503
Ангара (которая уже никуда не полетит в варианте 5П), Флакон, СЛС и Атлас тут авансом как недалекое будущее. А все остальные РН на пике - с реальными пилотируемыми полетами и ачивками за это. Энергия с блюдями не летала
288 370505
>>70498
Т.е ты хочешь сказать, что вот такую продукцию
http://www.fakel-russia.com/images/content/products/fakel_tkd_print.pdf

или такую
http://www.fakel-russia.com/images/content/products/fakel_spd_print.pdf
или вот http://www.niimashspace.ru/files/2016/Topliv_baki.pdf
и ещё много таких... продают только оптом? Как овощи. как флешки?

И стоят они, думаешь, пару тысяч?

А что я собрался делать со спутником дальше - кого ебёт? Я плачу бабки, за детали, за космический извоз на орбиту.
289 370506
>>70505

>А что я собрался делать со спутником дальше - кого ебёт?


Тебя ебет. Бизнес-план есть? Чем ты собрался взорвать рынок? Фотками кремля в шакальем разрешении? Или очередной спутник-ХУЯК, который будет Данон в ночном небе рекламировать?

>Т.е ты хочешь сказать, что вот такую продукцию


Думаю, да. Или мелкой серией, где мелкое >> 2 и > 20
290 370508
>>70506
Меня то ебёт и я знаю что буду делать с этим дальше. Если эксперименты будут удачные. А взрывать надо не рынки, а сознание людей.

И рыночек и бабки - совсем не главное в жизни

Т.е. эту продукцию у этих контор каждый день десятками покупают?) На такую продукцию эти конторы каждому заказу будут рады. Ради живых денег. Рыночек то решает)
291 370512
>>70506
Ты посмотри, сколько баков в год выпускают 20-60, да они охуеют, если я анон у них хоть пару в год закажет.
292 370513
>>70508

>Т.е. эту продукцию у этих контор каждый день десятками покупают?) На такую продукцию эти конторы каждому заказу будут рады. Ради живых денег. Рыночек то решает)



По твоим ссылкам речь о нескольких сотнях двигателей в год (наверное, суммарно по всему модельному ряду небольших электрореактивных двигателей). И они все производятся под заказ, они не производятся для выставления на полку магазина. Производителю не нужны твои пару сотен тысяч рублей, как я думаю, даже такому относительно небольшому производителю нужны заказы на миллионы и десятки миллионов. Своему клиенту они могут изготавливать штучный товар, но только при наличии действующего контракта и большой истории взаимоотношений или планов на будущее. Непосредственный производитель не занимается продажей, он дико не любит эту бумажную работу. Если есть "прокладка", которая у него покупает десятки двигателей и продает потом поштучно условному бизнесу вроде тебя - хорошо, но я думаю, в России таких компаний нету. А если и есть, то не все так гладко.

И вообще, хули тут спорить, если ты не омич и реально чего-то просчитываешь - позвони, да узнай. Не обязательно с двигателя начинать, который кубсату не нужен, набросай приблизительную компоновку спутника и предметно смотри, что потом из списка реально достать. Ты спросил - я ответил.
293 370592
Вот вы говорите саманта сё, саманта единственная вайфу.
Но ведь там ещё какая-то няша была, у которой есть ещё мини мем про стиральную машину. Волосы у которой торчком.
294 370594
>>67033 (OP)
что мешает колонизировать марс?
ультра ньюфаг
295 370596
>>70594
Гравитация, деньги и разреженная атмосфера Марса. Хотя технологии уже сейчас позволяют жить там.
d28c381c899301eb5d56a1634713231d.jpg188 Кб, 750x499
296 370600
>>70592
Полтора самантафага, как и любые фаги, заполняют пространство обсуждения на тему няшности космонавток объектом своей фагготрии, создавая у тебя впечатление, что тут считают няшной только Саманту. Но дело в том, что противостоять в битве вайфу самантафагу некому - в слоупочном разделе для аутистов всех больше волнуют вопросы космологии или аэрокосмоса, сверхмассивные чёрные дыры и узенькие щёлочки форсунок камер сгорания жидкостных реактивных двигателей.
А вообще в таком вопросе как няшность объективным быть малореально, потому что няшность определяется личным вкусом каждого.

>Но ведь там ещё какая-то няша была, у которой есть ещё мини мем про стиральную машину. Волосы у которой торчком.


Карен Найберг.
297 370623
>>70596

>разреженная атмосфера Марса


>>70596

>деньги


что мешает консолидироваться всем? Или это порашный вопрос?
Что насчет Венеры? Почему бы не решить вопрос с атмосферой?
298 370624
как держут связь с вояджером? Разве сигнал не должен искажаться излучениями от звезд и Солнца?
как его не разъебало в астероидном поясе?
299 370630
>>70410
Устарел. Обновлять - легче новый сделать, но дорого и не нужно
300 370631
>>70624

> как его не разъебало в астероидном поясе?


Довн уже 10 лет там летает и ничего. Меньше звёздные войны надо смотреть.

> как держут связь с вояджером?


DSN наса
301 370632
>>70495
Я уже давно мужчина! Просто искренне хочу помочь тебе с проектом;)
302 370633
>>70623
Пока мягкая посадка на его поверхность проблематична.

Кто и с кем будет консолидироваться? На Земле полно нерешённых вопросов и триллионы в Марс никто вбухивать не будет.
303 370649
>>70473
Илон, иди нахер!
304 370655
>>70513
А кто говорит про кубсат? Я хочу создать новый тип спутниковой платформы.
305 370676
>>70649
Я не он, меня зовут Антон.
306 370678
Почему существуют газовые гиганты? Почему гравитация не придавит весь газ к поверхности/ядру?
307 370679
>>70678

>Почему существуют газовые гиганты


Потому что существует гравитация и так уж получилось что вселенная заспавнивалась с огромным количеством водорода и гелия в первых эпохах после пиздеца большого взрыва.

>>70678

>Почему гравитация не придавит весь газ к поверхности/ядру?


Она это и делает, но газ копротивляется создавая йоба давление, которое вызывается электромагнитным отталкиванием атомов и молекул газа. У звёзд газ сжимает настолько, что атомы таки сталкиваются между собой и образуют ультра йоба ебовейшее давление от термоядерных реакций и много температуры ещё.
308 370680
>>70678

>Почему существуют газовые гиганты


Потому что существует гравитация и так уж получилось что вселенная заспавнивалась с огромным количеством водорода и гелия в первых эпохах после пиздеца большого взрыва.

>>70678

>Почему гравитация не придавит весь газ к поверхности/ядру?


Она это и делает, но газ копротивляется создавая йоба давление, которое вызывается электромагнитным отталкиванием атомов и молекул газа. У звёзд газ сжимает настолько, что атомы таки сталкиваются между собой и образуют ультра йоба ебовейшее давление от термоядерных реакций и много температуры ещё.
309 370681
>>70632
Мужчина, проститутки и коньяк то зачем?
Авдтоий 310 370684
Я вот не понимаю, откуда 350 кг сухого веса? На этой яхте?

https://dik.academic.ru/dic.nsf/ruwiki/1898167#.D0.90.D0.BD.D0.B0.D0.BB.D0.BE.D0.B3.D0.B8

Что там весит 350 кг? При объеме 1,21,20,6 м?

Движки ЭРД СПД-100 вместе 7 кг весят. Остальное что? Солнечные батареи, электроника и каркас? Из чего они блять сделаны? Из чугуниума?

и что значит "сухой" вес? Это вес с топливом, но без полезной нагрузки? Или это вес без топлива?

Основной вес - это топливо?
311 370698
>>70681
Ну что ты как маленький. Даже мне, мимокрокодилу, понятно. Это же очевидные элементы ведения дел в этой стране. Нужного человека нужно подмазать, начать с организации приятного досуга. А там уже, когда важный человек чувствует, что с ним обращаются соответственно его важному статусу, то сразу увидит, что его собеседник - не фуфел какой-нибудь, а ровный пацан, с которым и дела можно делать.
312 370721
>>70684
Сухой вес - это вес без топлива.
http://msiit.ru/NVlasov2005/htmls/proizvod_osn401.htm
image.png57 Кб, 1287x548
313 370722
>>70684
и без ПН, разумеется, если речь о платформе
DSD2Cm-W4AIZC.jpg large.jpg115 Кб, 960x637
314 370756
Объясните, зачем создавать ГЛОНАСС и Бейдоу? В чем смысл отдельной навигационной системы? Координаты же всегда одни и те же, не могут же американцы как-то их подменять для русских и китайцев? Или у США есть возможность сделать так чтобы координаты не выдавались каким-то пользователям? Ради риска такого стоит создавать свою навигационную систему? Или они еще для чего-то используются?
315 370760
>>70756
На истину не претендую.

Для определения своего местоположения, нужно знать координаты как минимум 3-х спутников GPS. Поскольку они летают по известным орбитам, а движение по орбите можно описать математически, то реальное местоположение спутников определяют не по факту (для этого нужен не приемник в 200 грамм, а радар в 200 тонн), а высчитывают на основе сигнала со спутников. Каждый спутник имеет на борту шибко точные атомные часы и передает с орбиты их показания. Приемник по сути ловит эти сигналы, считает чего он там наловил, определяет реальное положение спутников и считает текущую координату.

А теперь вопрос: как ты будешь считать свои координаты, если сигналы со спутника будут зашифрованы? И это не теоретическая задача, а практическая, сигналы по факты и так зашифрованы (просто с нами делятся ключом для дешифровки грубого гражданского сигнала, а военного, к примеру, нет) и технически эта возможность реализована.
316 370761
>>70756

> Или у США есть возможность сделать так чтобы координаты не выдавались каким-то пользователям?


Для военных, как я понимаю, там отдельный шифрованный канал. Отрубят гражданский канал, и чужие военные сосут хуй. Потому те военные, кто побогаче (и в НАТО), хотят такую же, но с блекджеком и шлюхами.
317 370764
Где именно на МКС стоит эта камера?
http://iss.stormway.ru/ru/cam2.html
image.png37 Кб, 1646x1023
318 370766
>>70764
А ты не видишь что ли?
HighDefinitionEarthViewingcameras1.jpg689 Кб, 4256x2832
319 370770
>>70764
The four cameras of the HDEV experiment are oriented in different directions and with different views relative to the ISS travel direction. They are in positioned, 1 looking forward, 1 looking nearly straight down, and 2 looking back. This provides several different viewing angles to the viewer.

The system is configured on the Columbus – External Payload Facility, which is a platform on the exterior of the European Space Agency's Columbus laboratory module where it is used to perform experiments to help NASA determine which cameras work best in outer space. The cameras are enclosed in a temperature-specific housing and exposed to the harsh radiation of space.

https://eol.jsc.nasa.gov/ESRS/HDEV/
https://en.wikipedia.org/wiki/High_Definition_Earth_Viewing_cameras
320 370773
>>70770
Спасибо.
321 370867
прочитал про dsn наса и нихуя не понял. Они используют мощные и чувствительные передатчики, но разве различные излучения в космосе не должны создавать помехи с искажениями?
322 370889
Внимание, гуманитарий в треде! Можете включать зеленый режим
Стори: когда я был пиздюком, отец мне часто читал всякие умные книжки про астрономию. От него мне и передался интерес к этой науке, но, в процессе обучения в школе, затем в шараге, после процесса отупления в армии и процесса реабилитации в вузике, (Хотя с вузом это неоднозначно) я полностью все забыл, абсолютно все. Если в вузике мне те остаточные знания, что я помнил с 5 лет о всяких созвездиях, черных дырах и.т.п помогали получать от других "черных дыр" то, что я тогда хотел, то после окончания работы и преобразования в офисный планктон с кредитошкодой я абсолютно все забыл. Но я хочу все вспомнить, почувствовать опять те ощущения, что я когда-то чувствовал, когда мне отец читал очередной раз свои умные книги про астрономию. Я хочу сам их читать. Но с чего начать? Книги, что он читал сгорели вместе с домом, в котором мы когда-то жили Какие-то научно-популярные журналы или даже журналы специализирующиеся на космической тематике? Посоветуйте мне хоть что-то, чтобы просиживать свое место в офисе мне хоть немного доставляло.
Продублирую этот вопрос еще и в /sci
323 370921
>>70867
Помехи можно отсеивать так как они идут лишь в одних частотах, а сигнал в других? К тому же они используют направленный сигнал и мощные антенны и тут просто нельзя спутать это дело с шумом. Как шум допустим это по мощности 1 или 2 единицы чего-то может заглушить сигнал, который по мощности 5-9 читается?
я вообще не разбираюсь в этому но думаю я недалек от истины
324 370923
>>70889
Могу лишь научпоп нашего времени посоветовать. На ютубе есть лекции Сурдина по астрономии, еще можешь Попова смотреть, на канале Архэ, их можно как аудиокниги слушать даже.
325 370924
>>70721
Ну так тем более, что там весит 350 кг?
14200440905880.jpg77 Кб, 500x499
326 370988
Поцоны, я слышал что НАСА выпускала альбом звуков космоса. Но я нихуя не могу его найти. Скиньте пожалуйста желательно в flac
327 370991
>>70988
Сам нашел, гуглится по "Nasa Voyager Recordings"
328 370996
Почему не сделают доставку на орбиту всякой хуйни с помощью воздушного шара в стратосферу, а там с помощью рейлгана уже выстреливать бочку с водой и пищей лунатикам на космические станции? Это же экономия топлива и стоит нихуя совсем.
329 371010
>>70996
Покажи мне рейлган, способный выстрелить ракетой хотя бы в пару тонн весом со скоростью хотя бы 3 км/с. А потом дирижабль, способный поднять эту ебанину в стратосферу.

Ни того, ни другого в данный момент вообще не существует даже на бумаге, а разработка, постройка и эксплуатация будет стоить просто сумасшедших денег. При этом экономия топлива невелика, а запускать подобная хуйня сможет только микроскопические грузы, а не 99% коммерческих аппаратов, массы которых от тонны начинаются.
330 371044
Знаю что ен по теме повпрос, но всё-равно спрошу: допустим я тону на корабле, и нахожусь на части корабля, которая затонет последней. Если я на ней буду находится в то время как она будет погружаться в воду - меня течения воды затянут к кораблю вниз и я утону или я останусь на оповерхности воды?
331 371051
>>70988
Пердеж космонавтов?
332 371062
>>71044

>я утону


Нет, ты сгоришь в атмосфере
333 371066
>>71044
http://myth-busters.ru/296-pravda-li-chto-tonuschee-sudno-sozdaet-silnuyu-voronku-kotoraya-mozhet-dazhe-uvlech-cheloveka-pod-vodu.html

Но не стоит воспринимать в лоб, они воссоздали только один набор условий. В других условиях их бы засосало и тому есть множество реальных историй.
334 371067
>>71066
А засосало бы так, что не выплыл? Просто я думаю если бы я на корабле плыл и он начал тонуть - что делать: прыгать в воду и отплывать или ждать и тонуть вместе с кораблём?
335 371068
>>71067
Надо моряков спрашивать, эти 100% знают и инструкцию тебе еще зачитают на этот случай. В фильмах почему-то все прыгают за борт в случае со спуском спас средств все понятно, и они там, кстати, всегда отгребают от места крушения. Может есть опасность, что из затонувшего корабля начнет топливо всплывать? Но с другой стороны вода холодная, зачем туда прыгать раньше времени?
336 371075
>>71068
Если ты в холодную воду прыгаешь без спассредств, то и прыгать-то в общем не обязательно, все равно пизда очень быстро.
337 371076
>>71075
Насколько быстро и болезненно?
338 371078
>>71076
При температуре воды 5-15°С опасным становится время пребывания в воде до четырёх часов. При температуре 2-3°С может стать смертельным пребывание более 15 минут. При температуре от 0 до -2°С летальный исход наступает после пяти-восьми минут. Известно также, что быстрота этого процесса зависит от физического состояния человека, от стойкости его организма к низким температурам, теплозащитным особенностям одежды и многого другого.

Гибель человека в холодной воде может наступить и раньше. Причиной становится шоковое состояние, так называемый «холодный шок», который развивается на протяжении первых пяти минут резкого охлаждения.

http://www.odnagdy.com/2010/11/blog-post_7231.html
339 371079
>>71078
Как тогда всякие люди зимой купаются?
340 371080
>>71079
Прям купаются вряд ли. Обычно бултыхаются секунд до 30 и назад под полотенце. Если сердечко молодое или имеется подобный опыт.
341 371083
>>71078

При температуре +28 смерть от переохлаждения наступает через сутки.
342 371205
Пару дней назад где-то в удмуртии что-то вспыхнуло. Подробности я прослушал. Вроде это было 10 января. Что это было и есть ли подробности\видео\предварительный отчёт?
343 371214
>>69344
поглащение ЦД объекта величиной с луну напрочь стирилизует всё пространство вокруг на световой год рентгеновским излучением
344 371229
>>71205
Газовый баллон в квартире? Ты о чем вообще?
345 371235
Почему такое распространение получили маргинальные форумы, паблики и каналы на Ютубе, в которых пропагандируется космических скептицизм. Смысл всех роликов, что в космос никто не летает, что всё снимается на зеленом экране и отмывается бабло. При том ролики получают тысячи лайков, сотни одобрительных комментов и минимум дизлайков. Что, блять, не так с этим миром?
346 371236
>>71235
космический скептицизм*
image.png229 Кб, 1642x668
347 371237
>>71235
Не знаю, анон, тем же вопросом давно задаюсь. Видимо тут надо копать в психологии масс + последствия общения через интернет.
348 371241
>>71235
Просто хотят выделиться из толпы, чувствовать причастность к небольшой группе людей, которые якобы знают настоящую истину
349 371245
>>67033 (OP)
Сап ануннаки. Посоветуйте плес пиндосские аналоги книг Чертока там или Каманина, где их диды делятся воспоминаниями о восходе космической эры СШП.
350 371271
>>67033 (OP)
Как сейчас ищут пришельцев? Тупо прослушивают все гармоники нейтрального водорода?

Пушо я так думаю, нужно самим начать излучать в просторы большого театра чтобы найти кого то.
Например, взять какой то дабстепчик и чистой синусоидой передавать его по частотной модуляции на частоте водорода. Это чтобы пройти тест на рандомные помехи, ибо непериодические колебания с простейшим шифром сразу покажут что это не радиошум.
И посылать эту хуйню по направлениям всех заметных объектов, которые видно с нашего куска камня. Например крабовая туманность. Очевидно что она довольно заметна и необычна.Поэтому её как мы, так и инопланетяне, будут часто изучать в телескопы. Поэтому нужно посылать сигнал по направлению туманости, чтобы цивилизация которая смотрит на туманность с противоположной стороны могли нас зафиксировать, и ровно в обратню сторону, чтобы ящерики находящиеся за нами тоже могли нас засечь.
351 371277
>>71271
Ты не понимаешь сути. Мы не хотим быть найденными всеми прочими формами жизни во вселенной, мы хотим найти сами и думать, что с этим делать. Аннунаки уже один раз прилетели, тебе мало что ли?
352 371278
>>71235
Результат экспериментов властей по оболваниванию быдла посредством информационных технологий с целью сделать массы более управляемыми. Очевидно же. Это такой РенТВ, только для этих ваших интернетиков.
353 371281
>>71235

Потому что это тонкота такая, такой вот тонкий троллинг. Быдло ведётся, кто поумнее ржёт. А тезисы о плоской Земле они довольно остроумные, я скажу.
354 371282
>>71235

>При том ролики получают тысячи лайков, сотни одобрительных комментов и минимум дизлайков


Ну а ты по многим подобным видео прошел и дизлайки проставил? Вот то-то и оно. Я не ставлю дизлайки чтобы это говно в ленте не отображалось. А че ты переживаешь? Скептицизм - это хорошо, это правильно. Особенно в нынешнем инфополе. Побочным выхлопом правда бывает ситуация, когда гуманитарии слишком увлекаются этим скептицизмом и лезут с ним в области, в которых не бельмеса не смыслят. Ведь в технической науке как? Надо не чье-то мнение вычитать, не статью авторитета, а объективную реальность воспринять, формулы, расчеты, техпроцесс. Конечно гуманитарии в это не могут. Не как что-то плохое. Если бы у меня по жизни душа лежала к философии и истории, а пять лет в вузике я учил бы латынь и перечитывал контроверсивные взгляды разных людей на исторические события и политико-социальные течения, то у меня бы мозг заточился на восприятие мнений и риторических выводов, а не на чтение чертежей, стройных рядов формул и написанных сухим языком инструкций.

Ведь и обратная стороны медали имеет место быть. Например история. Также важная и касающаяся всех наукану ладно, ладно, ненаука. Когда дохуя технари, пользуясь привычными механизмами математики и логики, нахватавшись обрывочных знаний из психологии, высерают в хистораче охуительные истории, допустим, про клерикальную политику Ришелье, основываясь на фактах его пристрастия к кошкам и кардинальском сане, явно намекающем на целибат, который, дескать, и был движущим шилом в заднице. От одного такого подхода к анализу реально толковые историки, коим я, например, совсем не являюсь, не меньше охуевают.

Тут опаснее, если какой-нибудь работник раскосого такой хуйней заболеет и станет плоскоземельцем. Это уже что-то уровня шизы. Таких вычислять надо. Но от форс-мажоров никто не застрахован. Ну хотят люди альтернативное виденье, жалко штоле? Чем бы дитя не тешилось, главное шоб на радикализировалось.
355 371283
>>71235

>Что, блять, не так с этим миром?


Все с ним нормально. 95% ведут себя так, как полагается 95%.
356 371286
>>71282

>увидел "форточку" на снимках с МКС


>пошел хуячить комментарии про плоскую Землю


>скептицизм



Скептицизм - Критически-недоверчивое отношение к чему-н., крайнее сомнение в чём-н.

Дебилизмом это в народе зовется. Когда человек, не может увидеть или хотя бы попробовать осознать картину целиком. Человек, запертый в ракушке своей картины мира из-за невежества. И это всё ничего, пока он безапелляционно не настаивает на собственной правоте. Вот это уже нарушение функций мозга и можно классифицировать как какое-то заболевание по МКБ.

Страшны не дети с их несформировавшейся психикой и не ограниченные в информации люди, которые накручивают лайки и комментарии под подобными видео. А страшны авторы каналов, которые постят эту хуйню. Сами не осилившие школьную физику, но считающие расстояние до звезд по параллаксу. И считают, что Солнце находится на расстоянии в Земле ближе, чем маршрут Пекин-Москва. Критику не воспринимают, проверить свои вычисления любой другой доступной методикой не хотят, ужаснуться от того, что они только что 1 формулой (чужой причем!, по которой как раз и находят реальное расстояние) уничтожили в прах всё научное сообщество, не могут. Эти же люди видят пузырьки воды в бассейне вместо мусора вокруг космических кораблей, они же воспринимают фото с рыбьего глаза как доказательства теории плоской Земли, они делают выводы о несостоятельности лунной программы по съемкам развивающегося флага и т.д.

И это нихуя не троллинг. Среди знакомых есть такие.
357 371293
>>71286

>Дебилизмом это в народе зовется.



Порвался, кек.
Ты поди и в безвредность ГМО веришь?
358 371295
>>71293
Настаивание на вреде ГМО, космический скептицизм, отрицание высадки американцев на Луне, ВИЧ-диссидентство, отрицание глобального потепления, существования динозавров, холокоста, ОТО, теории эволюции Дарвина - это признак дениализма и противоречит здравому смыслу!
359 371304
>>71295

>отрицание глобального потепления


Манюнь, отрицают не просто global warming, а human-made global warming, и это не фричество.
360 371307
hh
361 371312
>>71295

Ты поди зомбоящик вообще не выключаешь.
362 371316
>>71304
капиталисты будут отрицать что угодно ради маржи при сжигании ископаемых.
363 371317
>>71304
Мань, Нат Гео уже кучу документалок наснимал, как лоббисты в конгрессе заворачивают и мешают защищая всякие мобилы, про ручную прессу мердока, про тупарей трампов хуямпов не отличающих погоду от климата.
364 371318
Какой самый перспективный из разрабатываемых способ охлаждения космического корабля для межзвездного перелета?
365 371319
>>71318
магнитно-капельный?
если потери тела поборют
366 371320
>>71317

>документалок



О да, это весомый аргумент, в треде тупых вопросов-то.
367 371321
>>71318

В конечном счёте тепло можно выбросить только в виде излучения от радиатора.
368 371322
>>71320
По факту есть что сказать? Или лобби нефтяных гигантов не существует?
369 371323
>>71322

Как объединения-не существует. Их лобби на 99 процентов друг против друга, ну там например давайте ИГИЛ бомбить чтоб они нефти меньше продавали и всё такое. Потому что спрос на нефть продиктован запросами промышленности и уровнем развития науки и техники, уровнем развития культуры (культура это и есть промышленная и научно-техническая составляющая цивилизации, в данном контексте). То есть кричать что уголь хорошо а нефть плохо не получится, если нефть оказывается перспективнее угля то стоит просто переводить свой капитал туда. В противном случае твоё окружение останется со своими архаичными неконкурентными технологиями, а страна превратится в дыру по типу рашки.
370 371325
>>71319

>магнитно-капельный?


Это попытка узнать у меня правильный ли это ответ?
>>71321

>выбросить


Зачем его выбрасывать?
Ведь существуют адиабатные необратимые процессы в которых можно уменьшить энтальпию, увеличив энтропию системы.
371 371327
>>71325

>Ведь существуют адиабатные необратимые процессы в которых можно уменьшить энтальпию, увеличив энтропию системы.



Какие именно процессы ты предлагаешь?

>увеличив энтропию системы.



Ну предел есть же.
372 371331
>>71327

>Какие именно процессы ты предлагаешь?


Сколько мне заплатят за решение?

>Ну предел есть же.


У всего есть предел.
Достоинство системы в том, что ей не нужно ничего никуда излучать.
373 371332
>>71331
Тебе заплатят за щеку.
374 371333
>>71331

Всё ясно,изобретатель колеса в треде, все в волокушу.
375 371337
>>71332
>>71333

>заплатят за щеку


>изобретатель колеса в треде


Фрейд бы вас не пощадил.
Удачи вам с вашим летающим капельным говном в космосе.
Работать оно не будет, так как слабоумные дегенераты, которые его "изобрели", не учли одну небольшую деталь.
Но я вам ее не скажу - сюрприз будет.
376 371340
>>71337
Нам похуй.
377 371341
>>71318
Излучением от плазмафазного тела корабля, удерживаемого системой самозамкнутых магнитных полей?
378 371342
>>71337

>Работать оно не будет, так как слабоумные дегенераты, которые его "изобрели", не учли одну небольшую деталь.


>Но я вам ее не скажу - сюрприз будет.



И тебе удачи.
Я даже могу дорогу подсказать
http://www.findpatent.ru/catalog/6/105/

Посмотри, может тут есть уже твоё чудо-изобретение.
DMengines.png354 Кб, 747x535
379 371343
>>67120
Детонационники перспективны, но на самом деле они могут разве что в два -три раза облегчить пихло, за счёт урезания камеры сгорания и сопла с критическим сечением. Турбонаддувный агрегат остаётся, проблем управления фронтом волны выше крыши.
380 371344
>>67120
Ещё Цандер предлагал сжигать металлические части ракеты в двигателе. Фича в массовом совершенстве, это такая же важная характеристика ракеты, как и удельный импульс. Так что если придумать такое твердое топливо, чтобы горело равномерно, то можно даже с низким УИ на орбиту выйти в одну ступень. Но не нужно уже, человечество уже открыло ЖРД водорода кислород, этой реакции хватит всем. Только стартовая масса должна быть тонн под десять тысяч, чисто для окупаемости.
381 371346
>>71344

Можно баки из СВМПЭ делать, например.
15080491560730.jpg94 Кб, 713x563
382 371381
>>71340
<----
>>71342
Там его нет.
Как и некоторых других.
И не будет. Нет денег - нет мультиков.
>>71341
Магнитные поля конкретно в этой схеме вообще не нужны.
383 371401
>>71381

Ну тебе настолько не похуй, что ты прибежал сюда рассказать что ты изобрёл что-то такое что никто ещё не изобретал. Но при этом ты очень боишься рассказать, потому что спиздят. И запатентовать у тебя денег тоже нет, потому что ты нищеброд. Зато есть боевая картиночка.
384 371419
А поясните, пожалуйста, за суперэддингтоновскую аккрецию. Что вообще под этим подразумевается и как оно работает?
385 371438
Господа, летной погоды всем.
Объясните ситуацию с ТКС. Почему данное творение рук КБ Челомея не эксплуатируется больше? Утеряна технология производства? Дорого? Спускает он меньше, чем "Dragon", это да, но доставляет на тонну с лишним больше. Почему такая очередная несправедливость?
386 371439
>>71401
Этот проецирцющий еблан думает что его интеллектуальные права хоть как-то защищены в эпоху транснациональных корпораций. Найс.
Не ну я думал, что здесь совсем дегенераты сидят.
Но сегодня ты обновил дно.
387 371450
>>71439

---------------->/zog/
/law/<---------------
---------------->/po/
388 371451
>>71450
Читай сам свою парашу.
С таким "багажом знаний" неудивительно, что ты находишься на уровне развития собаки.
389 371452
>>71451
Памятка изобретателя с двача:
1. Ты гений, ты гениален.
2. То, что ты придумал ещё никто не придумал.
3. Не рассказывай свою идею, а то спиздят.
4. Не патентуй, а то спиздят.
5. Хвастайся как можно больше.
6. Называй всех гуманитариями, собаками и ебланами.
7. Используй боевые картиночки.
390 371484
>>67033 (OP)
Ньютонова пушка хоть теоретически возможна? Или все таки нельзя вывести на правильное наклонение одним выстрелом?
391 371485
>>71484
Срать на наклонение, ты перигей поднять не сможешь без второго импульса. А вот на отлётную траекторию запустить можно, это да.
392 371492
>>71484
Можно, достаточно просто вывести пушку на высоту будущего перигея и выстрелить точно с нужно места.
393 371563
>>71485

> ты перигей поднять не сможешь без второго импульса


А если как-нибудь по хитрому через гравитационный манёвр с Луной?
394 371659
Стукаются ли космонавты головой об потолок МКС когда пускают газы и подлетают вверх?
395 371663
>>71659
Тяга недостаточная.
396 371699
>>71663
А если срать не пристегиваясь к толчку?
397 371725
Как будут различать сигнал солнечного зонда на фоне Солнца? Я слышал что на него радиотелескопы вообще не направляют, а нужно же слушать зонд как-то?

https://www.youtube.com/watch?v=GoBPeCe4C2s
398 371727
>>71699
ну немного может и будет. хотя твои волосы в жопе тормозят говно, оно может вообще просто жидким пятном по жопе начать растекаться, или застрять в очке колбасой.
399 371752
>>71725

>Как будут различать сигнал солнечного зонда на фоне Солнца?



На фоне Солнца, а конкретнее в точке L1 прямо сейчас торчит 4 аппарата и как-то что-то никто не жалуется.
При том, что ты сам прислал ролик, в котором показана орбита, из которой ясно, что Зонд только в редких случаях появляется между Землей и Солнцем. Так что непонятно, в чем вообще проблема.

>Я слышал что на него радиотелескопы вообще не направляют


Ага, религия запрещает.
400 371764
>>71229
Вспышка баллона на 3 региона? Это сколько мегатонн?
401 371771
>>70889
Тоже мне проблема. Учебник по астрономии наверни.
402 371773
>>71322
Ты так говоришь что есть только лобби нефтянников которые к слову меняются. И если раньше это был частный бизнес, то сейчас их заняли гос.компании
А лобби зелёных не существует?
Все вот говорят что есть пропаганда за отсутствие ГП, но я лишь слышу пропаганду что дескать ты лично хика напердел много цо2. хотя вся автомобильная промышленность не жгёт столько сколько заводы и армия. И по этому тебе надо платить дополнительные налог на экологию, с товара или услуги.
403 371818
>>67033 (OP)
Что это за гибрид КамикадзеД и шаровераTM у вас на оп-пике?
404 371819
>>68806

>рождение навозного жука


Он жив ещё?
405 371823
>>71699
Тяга это масса*скорость+некий коэфициент расширения газов для ракетных двигателей. Говно из жопы выходит без напора и падает в толчок сугубо под своим собственным весом. Мощный дрыщ - да, тот может, наверное, создать небольшую тягу.
406 371838
>>71727

>хотя твои волосы в жопе тормозят говно


Бля, сук, чет в голосину с этого дульного тормоза.
Скажите честно, цивилизации и сложным структурам в любом случае пиздец? 407 371850
Проблема барьеров AlexAV для будущей цивилизации также существенна, особенно на больших промежутках времени:
Цитата: AlexAV от 10.01.2018 [15:08:58]
Цитата: Kaiserfrogling от 09.01.2018 [14:29:08]
Давай вспомним что именно эта тема делает на астрофоруме и каковы были предпосылки для ее создания?
Предпосылкой была гипотеза о разрешении парадокса Ферми через гипотезу "выеденного яйца" - после исчерпания природных запасов все внеземные цивилизации погибают, или хотя бы выживают но в такой форме которая неспособна к межзвездной колонизации.

Поддерживаю. Тема действительно посвящена больше долгосрочным перспективам, не обсуждать при этом текущие тенденции тоже нельзя, но избыточно концентрироваться на них здесь действительно не стоит. При этом обсуждая любые невозобновляемые ресурсы нужно всегда держать в голове, что абсолютно несущественно каковы их запасы, их истощение, в том случае если скорость с которой мы их добываем больше скорости с которой они пополняются естественными геохимическими потоками — неизбежно. Особняком тут можно было бы поставить только дейтерий морской воды (это если с термоядом...), но как следует из названия темы — мы этот случай не рассматриваем.

Тут стоит перечислить основные проблемы входящие в этот круг.

Как ни странно, но на первое место я бы поставил бы даже не замену нефти, газа и угля, а фосфор. Цивилизация может существовать без автомобилей, айфонов, интернета, но вод без хлеба она не протянет и нескольких месяцев. Никакая цивилизация.

И тут существует фундаментальная проблема — наше сельское хозяйство менее замкнуто по биогенам, чем естественные биогеоценозы. Уход биогенов с наших полей океан намного выше, чем для естественных лесов и степей. Если потеря азота, калия и серы — не такая большая проблема, то вот с фосфором — всё плохо.

Это вообще самый странный элемент биосферы, сравнительно редкий в земной коре, но при этом роль которого для живых организмов оказывается непропорционально высока. Элемент естественный транспорт которого на поверхности земли осуществляется только в одном направление — с суши в океанические глубины, а замыкание его круговорота осуществляется практически исключительно через медленные геологические процессы связанные с субдукцией океанической коры и выплавком магматических пород в материковой коре.

Сейчас мы компенсирует избыточную потерю фосфора с помощью удобрений, источником которых являются невозобновляемые запасы апатитов и фосфоритов. Но вечно так продолжаться не может. По большей части единственный выход здесь — как-то менять методы ведения сельского хозяйства так, чтобы потери биогенов с полей были не выше естественных фоновых. Но как это делать конкретно — не совсем понятно. Другим решением могло бы было бы быть — создание некого механизма транспорта фосфора из океана на сушу. Но как именно тоже не понятно (при этом нужно понимать, что тут речь не о добыче чего-то на морском дне, то что смывается с суши оседает там в рассеянной форме и добыть это напрямую практически невозможно, тут что-то другое нужно... возможно выращивание каких-то аквакультур или что-то ещё подобного...).

На втором месте — снова не нефть с газом и углём, а редкие элементы. Вообще все элементы кроме кремния, алюминия, железа, кальция, а так же элементов содержащихся в больших количествах в морской воде - натрия, калия, магния, хлора, серы, брома, главного элемента биосферы — углерода, и того что мы извлекаем из атмосферных газов, нужно считать не возобновляемыми ресурсами. Промежуточное положение тут занимают титан, марганец, цирконий, барий (доступны, но скорее всего в ограниченных объёмах).

Масштаб этого бедствия даже сложно оценить. Тут вылетают целые пласты технологии. Без жаропрочных сплавов просто невозможно построить ни ракетный двигатель, ни газовую турбину, что делать без нержавеющей сталей в огромном количестве областей вообще непонятно. Вылетают криогенные стали, большинство твёрдых сплавов. Сущая катастрофа получается в электронике.

На рециклинг особо надеяться не стоит. Потери при нём неизбежны (как в силу технологических, так и социальных факторов) и он может слегка оттянуть время, но никак не решить эту проблему. Придётся учиться жить только используя то что есть.

И только далее — энергетика. Тут очевидно — только возобновляемые источники энергии. Но нужно понимать, что если на «этикетке» написано «возобновляемый» - то он не обязательно возобновляемый, и наоборот если не написано — не значит, что не является возобновляемым. :) При этом при оценке того можно ли отнести источник к возобновляемым — абсолютно необходимо учитывать те материалы, которые нужны для его производства. Если необходимо что-то кроме элементов перечисленных выше — этот источник к возобновляемым отнести нельзя.

На сегодняшний день достоверно к возобновляемым можно отнести — гидроэнергетику, биотопливо в самом широком виде включая торф (проблему фосфора здесь можно опустить не по той причине, что она не важна, но потому, что если она не решается — проблемы энергетики будет последним, что нас будет интересовать :), когда вокруг банды каннибалов как-то не до энергетики...), ветроэнергетика (не вся, скажем как строить морские ветрогенераторы без нержавейки не понятно совершенно, но достаточное количество конструкций наземным ветрогенераторов указанному требованию удовлетворяют). Пожалуй всё.

Плюс к этому - под вопросом находятся ПЭС (приливные станции) и АЭС (при использование урана из морской воды АЭС — тоже (квази)возобновляемый источник энергии). Проблема коррозиестойких сплавов здесь будет стоять очень остро.

В варианте минимум (био- + ГЭС + ВЭС) — количество доступной энергии получается не очень большим. Значительно меньше, чем доступно нам сейчас, хотя больше чем в историческом средневековье (знание об электричестве позволяет использовать энергию ветряного и водяного колеса куда эффективнее, чем тогда).

Такой вариант, который условно можно назвать «средневековье с электричеством», мне и представляется кстати наиболее вероятным. Цивилизация в этом случае существовать может (впрочем это очевидно, в историческом средневековье она тоже ведь существовала), но от современной будет отличаться радикально.

Расширенный вариант (с АЭС) — подразумевает общество куда более похожее на индустриальное, по сравнению со «средневековьем с электричеством», но тут уж как получится... Вероятность такого варианта кажется существенно ниже...

Здесь кстати стоит отметить, что ряд вещей на которые ставят бирку «возобновляемого» на самом деле к возобновляемой энергетике нас вовсе не приближают, а отчасти даже удаляют. Ни в коем случае к возобновляемым нельзя отнести современные солнечные батареи или скажем электромобили. Просто из-за критической потребности в редких элементах. Дизельный двигатель куда ближе к принципам возобновляемой энергетики, чем электромобиль, как бы это странно не звучало. Дизельный автомобиль можно полностью сделать из чугуна, углеродистой стали и силумина, а в бак залить растительное масло. А вот как делать электромобиль без лития, кобальта и никеля — непонятно совершенно.

Четвёртое — минералогический барьер. Т.е. изменение минералогического состава верхней литосферы из-за длительного воздействия человеческой деятельности. Здесь конечно речь идёт не о ближайших столетиях, а о куда больших промежутках времени. Но в долгосрочной перспективе — это может быть очень важно.

Вы думаете гранитная щебёнка не может закончится? :) Зря так думаете.:) Гранит из литосферы вообще конечно исчезнуть не может, но вот количество выход магматических горных пород на поверхность уже куда как ограниченно. И выкапываем мы их куда быстрее, чем они туда поднимаются.

И таких аспектов очень много. Общая масса карбонатных пород в литосфере едва ли может измениться в результате любой деятельности человека. Но вот скажем изготовить известь из породы содержащей скажем 10% карбоната кальция едва ли возможно не сделав её золотой. А карбонатные породы, содержащие >90% карбоната кальция — ресурс уже не столь безлимитный.
Скажите честно, цивилизации и сложным структурам в любом случае пиздец? 407 371850
Проблема барьеров AlexAV для будущей цивилизации также существенна, особенно на больших промежутках времени:
Цитата: AlexAV от 10.01.2018 [15:08:58]
Цитата: Kaiserfrogling от 09.01.2018 [14:29:08]
Давай вспомним что именно эта тема делает на астрофоруме и каковы были предпосылки для ее создания?
Предпосылкой была гипотеза о разрешении парадокса Ферми через гипотезу "выеденного яйца" - после исчерпания природных запасов все внеземные цивилизации погибают, или хотя бы выживают но в такой форме которая неспособна к межзвездной колонизации.

Поддерживаю. Тема действительно посвящена больше долгосрочным перспективам, не обсуждать при этом текущие тенденции тоже нельзя, но избыточно концентрироваться на них здесь действительно не стоит. При этом обсуждая любые невозобновляемые ресурсы нужно всегда держать в голове, что абсолютно несущественно каковы их запасы, их истощение, в том случае если скорость с которой мы их добываем больше скорости с которой они пополняются естественными геохимическими потоками — неизбежно. Особняком тут можно было бы поставить только дейтерий морской воды (это если с термоядом...), но как следует из названия темы — мы этот случай не рассматриваем.

Тут стоит перечислить основные проблемы входящие в этот круг.

Как ни странно, но на первое место я бы поставил бы даже не замену нефти, газа и угля, а фосфор. Цивилизация может существовать без автомобилей, айфонов, интернета, но вод без хлеба она не протянет и нескольких месяцев. Никакая цивилизация.

И тут существует фундаментальная проблема — наше сельское хозяйство менее замкнуто по биогенам, чем естественные биогеоценозы. Уход биогенов с наших полей океан намного выше, чем для естественных лесов и степей. Если потеря азота, калия и серы — не такая большая проблема, то вот с фосфором — всё плохо.

Это вообще самый странный элемент биосферы, сравнительно редкий в земной коре, но при этом роль которого для живых организмов оказывается непропорционально высока. Элемент естественный транспорт которого на поверхности земли осуществляется только в одном направление — с суши в океанические глубины, а замыкание его круговорота осуществляется практически исключительно через медленные геологические процессы связанные с субдукцией океанической коры и выплавком магматических пород в материковой коре.

Сейчас мы компенсирует избыточную потерю фосфора с помощью удобрений, источником которых являются невозобновляемые запасы апатитов и фосфоритов. Но вечно так продолжаться не может. По большей части единственный выход здесь — как-то менять методы ведения сельского хозяйства так, чтобы потери биогенов с полей были не выше естественных фоновых. Но как это делать конкретно — не совсем понятно. Другим решением могло бы было бы быть — создание некого механизма транспорта фосфора из океана на сушу. Но как именно тоже не понятно (при этом нужно понимать, что тут речь не о добыче чего-то на морском дне, то что смывается с суши оседает там в рассеянной форме и добыть это напрямую практически невозможно, тут что-то другое нужно... возможно выращивание каких-то аквакультур или что-то ещё подобного...).

На втором месте — снова не нефть с газом и углём, а редкие элементы. Вообще все элементы кроме кремния, алюминия, железа, кальция, а так же элементов содержащихся в больших количествах в морской воде - натрия, калия, магния, хлора, серы, брома, главного элемента биосферы — углерода, и того что мы извлекаем из атмосферных газов, нужно считать не возобновляемыми ресурсами. Промежуточное положение тут занимают титан, марганец, цирконий, барий (доступны, но скорее всего в ограниченных объёмах).

Масштаб этого бедствия даже сложно оценить. Тут вылетают целые пласты технологии. Без жаропрочных сплавов просто невозможно построить ни ракетный двигатель, ни газовую турбину, что делать без нержавеющей сталей в огромном количестве областей вообще непонятно. Вылетают криогенные стали, большинство твёрдых сплавов. Сущая катастрофа получается в электронике.

На рециклинг особо надеяться не стоит. Потери при нём неизбежны (как в силу технологических, так и социальных факторов) и он может слегка оттянуть время, но никак не решить эту проблему. Придётся учиться жить только используя то что есть.

И только далее — энергетика. Тут очевидно — только возобновляемые источники энергии. Но нужно понимать, что если на «этикетке» написано «возобновляемый» - то он не обязательно возобновляемый, и наоборот если не написано — не значит, что не является возобновляемым. :) При этом при оценке того можно ли отнести источник к возобновляемым — абсолютно необходимо учитывать те материалы, которые нужны для его производства. Если необходимо что-то кроме элементов перечисленных выше — этот источник к возобновляемым отнести нельзя.

На сегодняшний день достоверно к возобновляемым можно отнести — гидроэнергетику, биотопливо в самом широком виде включая торф (проблему фосфора здесь можно опустить не по той причине, что она не важна, но потому, что если она не решается — проблемы энергетики будет последним, что нас будет интересовать :), когда вокруг банды каннибалов как-то не до энергетики...), ветроэнергетика (не вся, скажем как строить морские ветрогенераторы без нержавейки не понятно совершенно, но достаточное количество конструкций наземным ветрогенераторов указанному требованию удовлетворяют). Пожалуй всё.

Плюс к этому - под вопросом находятся ПЭС (приливные станции) и АЭС (при использование урана из морской воды АЭС — тоже (квази)возобновляемый источник энергии). Проблема коррозиестойких сплавов здесь будет стоять очень остро.

В варианте минимум (био- + ГЭС + ВЭС) — количество доступной энергии получается не очень большим. Значительно меньше, чем доступно нам сейчас, хотя больше чем в историческом средневековье (знание об электричестве позволяет использовать энергию ветряного и водяного колеса куда эффективнее, чем тогда).

Такой вариант, который условно можно назвать «средневековье с электричеством», мне и представляется кстати наиболее вероятным. Цивилизация в этом случае существовать может (впрочем это очевидно, в историческом средневековье она тоже ведь существовала), но от современной будет отличаться радикально.

Расширенный вариант (с АЭС) — подразумевает общество куда более похожее на индустриальное, по сравнению со «средневековьем с электричеством», но тут уж как получится... Вероятность такого варианта кажется существенно ниже...

Здесь кстати стоит отметить, что ряд вещей на которые ставят бирку «возобновляемого» на самом деле к возобновляемой энергетике нас вовсе не приближают, а отчасти даже удаляют. Ни в коем случае к возобновляемым нельзя отнести современные солнечные батареи или скажем электромобили. Просто из-за критической потребности в редких элементах. Дизельный двигатель куда ближе к принципам возобновляемой энергетики, чем электромобиль, как бы это странно не звучало. Дизельный автомобиль можно полностью сделать из чугуна, углеродистой стали и силумина, а в бак залить растительное масло. А вот как делать электромобиль без лития, кобальта и никеля — непонятно совершенно.

Четвёртое — минералогический барьер. Т.е. изменение минералогического состава верхней литосферы из-за длительного воздействия человеческой деятельности. Здесь конечно речь идёт не о ближайших столетиях, а о куда больших промежутках времени. Но в долгосрочной перспективе — это может быть очень важно.

Вы думаете гранитная щебёнка не может закончится? :) Зря так думаете.:) Гранит из литосферы вообще конечно исчезнуть не может, но вот количество выход магматических горных пород на поверхность уже куда как ограниченно. И выкапываем мы их куда быстрее, чем они туда поднимаются.

И таких аспектов очень много. Общая масса карбонатных пород в литосфере едва ли может измениться в результате любой деятельности человека. Но вот скажем изготовить известь из породы содержащей скажем 10% карбоната кальция едва ли возможно не сделав её золотой. А карбонатные породы, содержащие >90% карбоната кальция — ресурс уже не столь безлимитный.
408 371851
>>71850
Т.е. используя даже казалось бы обычные материалы — песок, известняк, глины для технологических нужды мы выбираем не абы какие, а с определенными свойствами. При длительном воздействии такой рода деятельности в верхней, доступной нам, части литосферы будет всё меньше минерального сырья удобного для технологического применения и всё больше его форм не годных ни для чего. Всё сложнее и дороже будет находить хорошие строительные материалы, материалы для производства керамик и стекла.

Данная проблема плохо изучена и в общем относится к довольно отдалённому от нас периоду. Но её воздействие на цивилизацию также может быть гигантским.

Ну и наконец — важный вопрос это ограничения и лимитирующие факторы человеческой деятельности. Цивилизация существующая на возобновляемых ресурсах тоже не может расти неограниченно. Её рост естественным образом будет упираться в их конечную скорость возобновления. И тут уместно вспомнить про классическую бочку Либиха. Рост любой популяции всегда ограничен одним наиболее редким ресурсом.

Соответственно возникает очень важный вопрос — какой конкретно ресурс будет ограничивать цивилизацию в этом случае. И на каком уровне он будет ограничивать размер численности населения и объёма экономической деятельности.

Впрочем чтобы на него ответить — нужно сначала ответить на вопрос как будут решены проблемы четырёх выше описанных барьеров...
409 371853
>>71850
>>71851
Если не одна маня сифи\космоопера\фентези хрень для нормального БАФА развития цивилизации невозможна, то в перспективе на неограниченные промежутки времени... Пиздец таки настанет причем не только сложным структурам, но и даже кваркам. Правда с ними уже мутновато, ибо проблема конца вселенной уровня "откуда вообще большой взрыв" и тому подобных.

Ну и паста в тему. Ахтунг грубо написанная, даже щитпостингом воняет, но если тщательно подумать то... мда:
Нихуя среад

Вот когда я сдохну, от меня останется огромная куча биомассы, она будет гнить и вонять истончая наипротивнейший аромат её будут жрать бесчисленные формы жизни, которые будут в неё же потом срать говном, которое будет вонять ещё больше, потом другие формы жизни будут жрать эти конвертировать их в говно и срать этим говном сверху той кучи говна что от меня останется, в итоге от меня останется лишь мерзкая болотная жижа, которая будет протекать сквозь землю и впитываться силикатными соединениями, потом это говно всосут мхи, лишайники и бесчисленная микробиота в почве, формируя из этого говна свои ткани. Итак будет продолжаться снова и снова, меня будет перерабатывать в говно, а из говна делать новые формы жизни, потом эту траву сожрёт нечто сложное и потом снова переработает в говно и посрёт тем говном, что останется от меня, так будет продолжаться сотни миллионов лет, я буду перерабываться в говно и этим говном из меня будут срать, как делали миллиарды лет и до этого. Потом поверхность земляшки раскалится настолько что я стану частью океана кипящей магмы, постоянно истончающей зловонные газы и сжигающей всё до чего прикоснусь, океан кипящего силикатного говна сжатый колоссальным атмосферным давлением, после миллиардов лет гниения и разложения, только вообразите насколько это прекрасно... Потом сраная звезда проебёт весь водород и всё обледенеет, но скорее всего земляшка ебнится об карлана и то говно что от меня останется сожмёт невозможной гравитацией. Потом настанет смерть тепла или ещё какое нибудь говно вроде биг разрыва\биг схлопа или фазового распада и в конце всё застынет в пустоте где нет не одной флуктуации и настанет пиздец всему! Останется лишь изначальное нихуя которое сконвертировалось во всё говно! Восславим же говно существовавшее вне времени и пространство, ведь соткано всё и ничто из него! ГОВНО! ГОВНО! ГОВНО! ГОВНООООООО
409 371853
>>71850
>>71851
Если не одна маня сифи\космоопера\фентези хрень для нормального БАФА развития цивилизации невозможна, то в перспективе на неограниченные промежутки времени... Пиздец таки настанет причем не только сложным структурам, но и даже кваркам. Правда с ними уже мутновато, ибо проблема конца вселенной уровня "откуда вообще большой взрыв" и тому подобных.

Ну и паста в тему. Ахтунг грубо написанная, даже щитпостингом воняет, но если тщательно подумать то... мда:
Нихуя среад

Вот когда я сдохну, от меня останется огромная куча биомассы, она будет гнить и вонять истончая наипротивнейший аромат её будут жрать бесчисленные формы жизни, которые будут в неё же потом срать говном, которое будет вонять ещё больше, потом другие формы жизни будут жрать эти конвертировать их в говно и срать этим говном сверху той кучи говна что от меня останется, в итоге от меня останется лишь мерзкая болотная жижа, которая будет протекать сквозь землю и впитываться силикатными соединениями, потом это говно всосут мхи, лишайники и бесчисленная микробиота в почве, формируя из этого говна свои ткани. Итак будет продолжаться снова и снова, меня будет перерабатывать в говно, а из говна делать новые формы жизни, потом эту траву сожрёт нечто сложное и потом снова переработает в говно и посрёт тем говном, что останется от меня, так будет продолжаться сотни миллионов лет, я буду перерабываться в говно и этим говном из меня будут срать, как делали миллиарды лет и до этого. Потом поверхность земляшки раскалится настолько что я стану частью океана кипящей магмы, постоянно истончающей зловонные газы и сжигающей всё до чего прикоснусь, океан кипящего силикатного говна сжатый колоссальным атмосферным давлением, после миллиардов лет гниения и разложения, только вообразите насколько это прекрасно... Потом сраная звезда проебёт весь водород и всё обледенеет, но скорее всего земляшка ебнится об карлана и то говно что от меня останется сожмёт невозможной гравитацией. Потом настанет смерть тепла или ещё какое нибудь говно вроде биг разрыва\биг схлопа или фазового распада и в конце всё застынет в пустоте где нет не одной флуктуации и настанет пиздец всему! Останется лишь изначальное нихуя которое сконвертировалось во всё говно! Восславим же говно существовавшее вне времени и пространство, ведь соткано всё и ничто из него! ГОВНО! ГОВНО! ГОВНО! ГОВНООООООО
410 371856
>>71850
Да, к 1950 году Москва будет погребена под горами лошадиного навоза.
Тогда же население Российской Империи превзойдёт полмиллиарда.
1352749649983.jpg14 Кб, 280x210
411 371858
>>71492

>достаточно просто вывести пушку на высоту будущего перигея


Содомит!!!
412 371871
>>71850
Сам же говорит что до начала этих проблем сотни лет.
За пару веков можно таки научиться добывать полезные ископаемые не на Земле
413 371877
>>71419
Ну бамп же, ананасы. Я же знаю, тут есть разбирающиеся люди.
Сверхкритическая аккреция.png14 Кб, 1001x855
414 371887
>>71877
>>71419
Есть некий предел светимости для всех звезд, при котором излучение компенсирует гравитацию: предел Эддингтона. Грубо говоря, когда свет мешает выпадать веществу на тело. Если достигнут эддингтоновский предел, то звезда находится в стабильном состоянии, она не сжимается и не расширяется. При превышении этого предела свет из недр звезды начинает срывать внешнюю оболочку, что вызывает ебовейший звездный ветер.

Что касается сверхкритической (суперэддингтоновской) аккреции, то тут все почти тоже самое. Если свет, излучаемый выпадающим на компактный объект (ЧД или НЗ) веществом, превышает этот предел, то вещество уже не может выпадать на тело, пока мощность излучения не спадет. Тем не менее встречаются компактные объекты, у которых этот предел явно и значительно превышен. Это возможно при неоднородном излучении. Например, когда часть вещества закручивается в сверхмощных магнитных полях и выбрасывается через полярные струйные течения (джеты). При том, если джет направлен в сторону наблюдателя будет превышаться предел Эддингтона. То есть в ебало прилетает более мощное излучение, нежели то, если бы ты смотрел на объект сбоку.
415 371889
>>71887
А, т.е. имеется в виду что то, что превышает предел Эддингтона не идёт на увеличение массы объекта а просто отстреливается в сторону. Ясно, спасибо большое, картинка в голове улеглась.
416 371891
>>71889
Имеется ввиду, что этот объект излучает больше, чем может/должен.
Данная модель объясняет противоречия. То есть объясняет откуда берется больше излучения, чем может быть в принципе.
417 371907
>>71853
Годная паста, схоронил
418 371940
>>71563
Можно. Но зачем?
419 371964
Поясните дебилушке про ИТЭР, ведь он противоречит занонам физики, ведь нельзя проихзводить больше потребляемой энергии. Или я не понял что-то?
420 371966
>>71964
ТЯР:
энергия на выход = энергия синтеза - затраченная энергия
Погугли вообще что такое термоядерный синтез
421 371975
Чтоб такого почитать из тех.лит , чтоб начать строить из себя ракетного инженера?
422 371981
>>71975
Гайды по Kerbal Space Program.
423 371987
Тупой вопрос не по теме: где можно позадавать тупых вопросов на разные темы?
424 371988
>>71987
В /b
425 371989
>>71987
В sci есть аналогичный тред тупых вопросов
sage 427 372029
>>71987
Есть ещё тред "очень тупых вопросов" в сифиче.
428 372067
>>71964

>Или я не понял


This
429 372068
>>71987
Задавай здесь, всё равно не прогонят
430 372078
>>67033 (OP)
Если всё вращается вокруг тела с наибольшей массой, то может ли это означать то, что всё крутится вокруг огромной чёрной дыры? Пизда ли всему, рано или поздно?

Что такое космос - чёрное пространство, в котором всё вертится? Был ли он всегда (это как?) или же неким образом появился? А если появился, то что было до него? Ничего? И из ничего сотворилось что-то?

Как много нужно условий, чтобы зародилась жизнь? Сам факт существования экзопланет доказывает существования на ней жизни в той или иной форме?

Какой формы вселенная и почему?

Если вселенная зародилась от большого взрыва, то можно ли говорить о том, что все потенциально населённые жизнью планетки начали своё развитие примерно одновременно? Это к тому, что возможно нигде нет эдемских условий развития общества, чтобы уже сейчас существовали активно бороздящие просторы чёрного цивилизации.

Последний тупой вопрос : допустим, жизнь ещё где-то есть. В таком случае, её форма и максимальный потенциал зависят от природных условий? Можно ли представить заснеженную горную планету, где скачут обросшие белым мехом "полулюди"-"полугорные козлы"?
431 372082
>>72078

>всё крутится вокруг огромной чёрной дыры


Что всё?

>Пизда ли всему, рано или поздно?


Неизвестно.

>Что такое космос - чёрное пространство, в котором всё вертится?


В самом широком понимании - мироздание вообще, Вселенная. В узком смысле все, что вне той или иной среды. Например, вне атмосферы Земли. Грубо говоря - все, что лежит вне небесных тел, за их атмосферами.

>Был ли он всегда


Нет.

>или же неким образом появился?


Некоторым образом появился. Господствующая теория - теория большого взрыва.

>Ничего?


Не совсем. Было некоторое начальное состояние, о свойствах которого ведутся споры. Окончательного ответа нет и возможно, никогда не будет.

>Какой формы вселенная и почему?


Неизвестно.

>можно ли говорить о том, что все потенциально населённые жизнью планетки начали своё развитие примерно одновременно?


Нет, нельзя. Есть некая абстрактная точка, до которой зарождение планет не было возможным, так как не было веществ тяжелее лития. Должно было пройти какое-то время для накопления во Вселенной достаточного количественного и качественного разнообразия элементов, для формирования планет.
При этом вот это возможно:

>возможно нигде нет эдемских условий развития общества, чтобы уже сейчас существовали активно бороздящие просторы чёрного цивилизации.



>В таком случае, её форма и максимальный потенциал зависят от природных условий?


Безусловно, наличие тех, или иных качеств и существования вида вообще возможно только при соответствии его строения и функций условиям существования. Случайные мутации, которые очевидно не дают преимущества в выживания в условиях могут сохраниться, однако те, что явно мешают выжить будут пропадать, вместе со смертью носителя и его потомков, если такая мутация ими унаследуется. В примере полукозлов-полулюдей не будет лысых, так как они замерзнут, не будет с черным мехом, потому что их съедят хищники, так как их будет очень заметно на белом снегу. И так далее.
432 372085
>>72078

>Если всё вращается вокруг тела с наибольшей массой


Если. А оно не вращается. Всё вращается вокруг общего центра масс, просто это не так заметно если одна масса на порядки меньше другой. Но на вращение того же Солнца влияет даже самая маленькая песчинка на Альфе Центавра. Стало быть, нет, не может означать. Только если за тело не принять саму Вселенную — но даже такое будет трудновато доказать.

>Как много нужно условий, чтобы зародилась жизнь?


Какой-нибудь материал, который потенциально может без особых проблем соединяться в сложные соединения. Что-нибудь, что может колбасить этот материал во все стороны так чтобы он рандомно скручивался в какие угодно структуры что физическим, что химическим путём. Агрессивная внешняя среда. Дохренища времени, просто настолько дохренища, что для человеческого мозга даже миллионная доля была бы огромным промежутком времени. Всё. Если интересует Земля, то у нас это были: Углерод. Вода и течения плюс различные условия для протекания хим. реакций. Различные природные явления вроде слишком сильных течений, молний, подводных вулканов и прочей злой природной хераболы, а впоследствии и другие организмы. Миллиарды лет. 4/4.

>В таком случае, её форма и максимальный потенциал зависят от природных условий?


Её ВСЁ зависит от природных условий и случайностей. Более приспособленные организмы выживают чаще и поэтому всё живое постепенно подстраивается под текущие условия, следовательно, эти условия и оказывают решающий вклад.
433 372108
>>72085

>Всё вращается вокруг общего центра масс



Даже на масштабах сверхскоплений галактик ничего уже друг вокруг друга не вращается, это уже гравитационно несвязанные структуры. Расширение пространства пересиливает и скопления галактик постоянно удаляются друг от друга, не делая вокруг чего бы то ни было никаких оборотов.

На крупномасштабной структуре вселенной, которая выглядит как пена, тоже никакого вращения нет, только малоподвижные локальные уплотнения.
434 372161
>>72108
Ну какое-то ускорение друг другу они всё же должны придавать, ибо их гравитационные волны пока друг до друга доходят, пусть и с сильным пингом и даже учитывая, что именно нынешние скопления уже вряд ли смогут связаться друг с другом из-за расширения.
435 372162
>>70592

>Волосы у которой торчком.


Ты только что любого человека на орбите с длинными волосами.

>>70600
Два чаю.
самантафаг
436 372163
>>70760

>Каждый спутник имеет на борту шибко точные атомные часы


Как эти часы работают?
437 372164
>>71277
Бритва Хэнлона, братан.
Никогда не приписывайте злому умыслу то, что вполне можно объяснить глупостью.
438 372165
>>72164
Промазал. Адресовано сюда >>71278
439 372166
>>71283
Самое примечательное в быдле это то, что оно всегда не ты.
reece.JPG27 Кб, 504x415
440 372168
>>71343

>Турбонаддувный агрегат


>Турбонаддувный


Как с умным видом спиздануть хуйню и не подать виду.
441 372169
>>71438
Бамп хорошему вопросу.
442 372177
>>72169
пояснили в роскосмос-загоне
12648699[1].jpg401 Кб, 503x719
443 372192
>>71235

> Почему?


Кто-то разыграл эту карту
444 372267
А космонавты на МКС вообще дрочат? И если да, то как? Это же уединиться надо где-то, и потом ещё чтоб ничего не заляпать, как-то надо утилизировать всё. А если не дрочить, то потом в один прекрасный день случится ночной сюрприз, проснёшься потом - а вокруг капли кончи летают.
445 372272
>>72267
На флоте месяцами не дрочат, никаких особых проблем кроме острого желания запихнуть бойца в чью-нибудь норку. Некогда, негде и обстановка не позволяет.

А на МКС мужики 40-60 лет, даже если случатся поллюции, дальше трусов они никуда не улетят.
0oMhDdnZScA.jpg37 Кб, 684x386
446 372277
Космический вакуум, как известно, не прямо вот совсем вакуум, а просто очень-очень разряженное пространство. И хотя около звёзд, планет и прочего эти молекулы уплотняются, в целом по вселенной приблизительно одинаковый уровень плотности вещества.
Так вот, берём и переносимся в такую часть вселенной где на протяжении дохуя миллионов и миллиардов км стабильное и равномрное распределение частиц в пространстве, можно сказать, до однородности. Теперь берём и помещаем в это ничего какой-нибудь обычный соверменый военный самолёт, который, силой Господа нашего Бога и воображением анонимусов со спейсача, способен разгоняться до околосветовой, световой или сверхсветовой (закроем глаза на то, что это невозможно и на эффекты массы и времени для наблюдателя внутри самолёта в этот момент). Так вот, на какой скорости этот самолёт получит плотность вакуума достаточной, чтобы частицы, встречаемый самолётом, встречали столь же плотно и часто, сколь и самолёту на Земле встречаются молекулы воздуха в атмосфере при полётах? Как быстро надо двигаться самолёту в космосе, чтобы снова начали работать законы аэродинамики?
447 372280
>>72277
На таких скоростях уже вроде как частицы не будут взаимодействовать с самолётом как атмосферные, они будут "ударяться" об него с бешеными энергиями если им таки удастся быть поглощёнными материей самолёта или пролетать его на сквозь с минимумом взаимодействий, фактически эти частицы будут либо пролетать его на сквозь либо пытаться расщепить его материю тем самым вероятно оказывая ему сопротивления и замедляя его скорость, для аэродинамики нужны куда более мягкие касания.

Если ты разгонишь его до световой то придашь ему бесконечную энергию и всё сколлапсирует в YOBA черную дыру с сингулярностью "по всюду" тем самым заставив известные физические законы формирующие пространство время пойти по пизде и сформировав некие новые, где например бесконечная энергия будет взята за энергию... ну например как сейчас энергия одного электрона и т.п. тут в общем маня \b\ начинается никем не анализируемое и не проверяемое.

Если ты берёшь сверх световую то это вообще пушной зверь для физических законов, в лучшем самом гипотетически твёрдом случае у тебя самолёт в тахионную материю сконвертируется с хз какими эффектами ибо не понятно допускает ли наша физика существования тахионов или нет. Либо опять же заставишь физику или во всей вселенной или только в рамках самолёта пойти по пизде и либо опять будет YOBA черная дыра с сингулярностью по всюду либо крайне специфический фазовый распад вакуума, либо новый большой бабах внутри этой же вселенной либо ещё какая нибудь НЁХ уровня маня \b\. И вроде бы тахионы должны постоянно перемещаться в прошлое..
448 372282
>>72277
Давай посчитаем. В межгалактическом пространстве имеем примерно один атом водорода на литр вакуума, это будет совсем не густая среда, плотностью 1.66×10-24 кг/м³. Воздух у нас имеет плотность 1.265 кг/м³. Самолет может летать при 100 км/ч, это почти 30 м/с то есть 30 метровый столб воздуха с основанием 1 м2 преодолевается за 1 секунду, в этом столбе содержится воздуха на 30×1.265 килограммов. Какой длины будет столб межгалактического вакуума такой же массы? Делим массу на плотность и получаем объем - он численно будет равен длине и искомой скорости поскольку у нас основание и время единичны. Это 2.28×1025 м/с или превышение скорости света в 7.62×1016 раз или расстояние в тысячу раз превышающее расстояние до галактики Андромеды за секунду. Как видишь скорости запредельные даже если закрыть глаза на басни Эйнштейна.
449 372283
>>72277

>в целом по вселенной приблизительно одинаковый уровень плотности вещества



Да прям. В среднем по вселенной — меньше 1 атома на кубический метр (примерно в 10^25 раз менее плотная среда, чем воздух на уровне моря), но даже на самом крупном масштабе вселенная не однородна, а состоит из этаких галактических нитей, где почти вся масса, а между ними войды, где средняя плотность в десятки раз меньше.

Но если об этом не думать, а надо просто увеличить кажущуюся плотность среды в 10^25 раз, то и «кажущуюся» скорость нужно увеличить во столько же раз. Поскольку скорость света примерно в миллион раз выше скорости самолета (3×10^8 м/с против 300 м/с), нам нужен лоренц-фактор порядка 10^19, что достигнется при скорости порядка √(1 - 1/(10^19)^2) = ~0.99999999999999999999999999999999999999c (отличие в тридцать восьмом знаке).

Разумеется, не то что на такой скорости, а даже и на 0.9c набегающие атомы самолет моментально уничтожат, так что придумывай менее омский способ полетать между галактиками.
450 372290
>>72283

>0.99999999999999999999999999999999999999c


Ну на счет этой скорости то вероятно проблемы будут, но http://antihydrogen.livejournal.com/46578.html тут пишут что 0.9 таки относительно терпимы.
451 372295
>>72282
Может, унутре галактики попробовать? Там 1000 атомов на кубический сантиметр
452 372296
>>72295
Можно подумать, ты внутри галактики со свечёй стоял и атомы считал. может их там 1500 или 900 на кубосантиметр. Так что не пизди и не вводи в заблуждение, умник блядь.
453 372297
>>72296
Усредненное значение посчитать можно, пошел нахуй
tumblro11513ZVLB1r20fq5o11280.jpg674 Кб, 1280x1280
454 372340
https://psv4.userapi.com/c834703/u32203143/docs/d17/28458af399f9/Razmery_zvyozd_i_planet.gif?extra=hJ5eOPTXi_esCsPam5g3G89eiULDA8eVNDU664Dhu5PfJvc3wQBoVaye_HPEceY17go5WMoFKDaD557BX3Wbb1-k4cMTKSFOkGIyUXClYaFJxFTKL5oX1-iNIOHXOM8ZLjCPy1TY6NBO3Q
Шо це таке анон за звёзды на первых позициях на гифке? Я думал. таких звёзд не бывает и что мельчайшая по объёму - это 2МАСС, которая чуть меньше Юпитера.
455 372343
>>72340
Дали человеку возможность нормально прикреплять картинки, нет, надо обязательно обосраться.
456 372344
>>72290
А разве набигающие атомы будут быстрее с. По моему время на них как раз будет давить, чем на самолёт.

Алсо. По идеи самолёт не развалится а станет более хрупким, и в принципе держать форму будет. И при такой плотности происходить это будет не мгновенно. И прежде чем на нём появится царапинка, экипаж уже сгорит в рентгене.
457 372349
>>67033 (OP)
Поясните за фото с поверхности Венеры. Сняли на цифру и передали прямо в направлении земли? Или на пленку и сразу внутре венерохода оцифровали?
458 372351
>>72349

>Сняли на цифру и передали прямо в направлении земли?


This, ну сорт оф. считай, это была телевизионная трансляция фотографий с Венеры.
459 372353
>>72351
А на такие большие расстояния и такой точный прицел на Землю, длинные или короткие волны использовались? Или оно веером "примерно в ту сторону" спамило?
460 372357
>>72353
Сигнал передавался не прямо на Землю, а сначала со спускаемого аппарата на оставшийся в космосе ретранслятор, а оттуда уже узконаправленной антенной на Землю. Использовался сантиметровый и дециметровый диапазон.
461 372359
>>72357
Круто, спасибо.
Получается, подгадали, когда обе планеты будут близко?
462 372363
Очень наверное тупой вопрос, но я все же спрошу.
Что, блядь, такое квазар?
Сколько читал википедию, так и не вкурил. Это такая черная дыра?
463 372366
>>72363
Типа того. СверхМассивная Чёрная Дыра в центре галактики, которая активно жрёт говно вокруг себя и активно же излучает в окружающее пространство.
464 372377
>>72267

Короч такая хуйня, что в невесомости хуй не стоит.

>>72272

На флоте сейчас дрочат или ебутся, а раньше друг дружку пялили или юнг, или проституток портовых.
465 372378
>>72357

>Сигнал передавался не прямо на Землю, а сначала со спускаемого аппарата на оставшийся в космосе ретранслятор


А на какой ракете запускалось? Что-то уникальное типа Сатурна5? Сейчас так можно сделать?
466 372379
>>72377

>На флоте сейчас дрочат


Могу оспорить, я в армейке когда служил, даже без брома от стресса был так заебан, что про секс даже мыслей не было.
467 372383
>>72378
На обычных Протонах с разгонным блоком, там не особо тяжелые зонды были, всего несколько тонн весом.

Роскосмос обещают к 2026 году Венеру-Д, которая продержится на поверхности несколько месяцев, а не пару часов, но дадут денег или нет, еще хуй знает.
468 372399
>>72377

>в невесомости хуй не стоит


Кто долбоёб? Ты долбоёб.
469 372414
>>72399

Нет ты, идиотина тупорылая. Ссу тебе в ебало. А ты ротешник открывай и сглатывай, маня.
470 372416
Поясните за скорость света. Свет летит от звезды в нашу сторону. Мы вместе с Землей летит в сторону этой звезды. Почему скорость света в итоге не превышена?
471 372421
>>72343
Обосрался ты, ибо у борды есть лимит на вес гифок
472 372422
>>72272
С чего ты взял, что на флоте не дрочат? Я хоть и сухопутная крыса, но дрочил в армии немало.
473 372423
>>72363
Все эти квазары, блазары, залупазары - разные термины для одних и тех же объектов. Которые суть чд в центрах голактик, разнонящие говна вокруг себя до релятивистских скоростей. Разница лишь в том, какой стороной эти зары повернуты именно к нашей земляшке.
474 372424
>>72379
А у нас служили тни на контракте, на которых тайком спускала вся часть срочников.
475 372425
>>72383

>Роскосмос обещают к 2026 году Венеру-Д, которая продержится на поверхности несколько месяцев, а не пару часов, но дадут денег или нет, еще хуй знает.


На Ангару им дают деньги много лет подряд, а просто повторить подвиг с фотокамерой не дают. Пиздец.
476 372428
>>72416
Представь, что ты бежишь по дороге, которую растягивают. И как сильно ты бы не бежал, растягивать дорогу будут сильнее. Вот и скорость света так бежит.
Суть спейсача.PNG604 Кб, 1562x759
477 372429
478 372453
>>72340
404
Почему нельзя было найти оригинал на тытрубе и выложить его, например?

>2018


>гифки


Это ж пиздос.
479 372454
>>72416
Потому, что скорость - это расстояние на время.
При релятивистских скоростях меняются расстояние и время. Как бы ты ни разгонялся, твоя секунда станет иной, твой метр тоже изменится. В итоге ни ты не сможешь достичь с, ни свет не будет иметь иную скорость.
Есть много довольно удобоваримых роликов на эту тему у PBS SpaceTime и ХуеВиСоса, но ты, наверное, английский не знаешь.
480 372456
>>72416

Скорость света всегда постоянна, не зависимо от скоростей источника и приёмника. Это было обнаружено экспериментально и легло в основу специальной теории относительности.
1516479722410.png33 Кб, 301x301
481 372461
Почему метеориты всегда приземляются в кратеры?
482 372462
>>72456
Я слышал что скорость света в среде меньше чем скорость нейтрино.
483 372465
>>72462
В среде не только нейтрино обгоняют. Гугли свечение Черенкова
484 372469
>>72461
Мироздание хорошо играет в гольф.
485 372470
>>72469
Да и в кости неплохо.
486 372472
>>72462
пиздят смотря в какой среде
487 372487
>>72462

В среде меньше, верно. Коэффициент преломления и показывает во сколько раз меньше. То есть если к-т преломления у стекла 1.6 то в 1.6 раз меньше.
Я вакуум имел в виду. Если источник приближается к наблюдателю, то растёт энергия фотона, то есть длина волны падает. Если удаляется, то растёт. Это называет эффект Доплера. Она наблюдается и со звуком. Например когда мотоцикл едет навстречу, то он веееееееееееее а когда проезжет и удаляется, то ееуууууууууууууу. Скорость звуковых волн почти одинаковая для всех частот, так же как и скорость света. Это и наводила на размышления о свете как колебаниях эфира.
488 372490
Анон, посоветуй каналов на тытрубе про рокет саенс, но чтобы понятно и интересно для быдла было. Можно пиндосские.
489 372493
>>72462
А в четверге она меньше, чем у черепахи Зенона.
490 372501
>>72490
Скотт Мэнли
491 372534
>>72501
Он наше всё, но довольно выборочно и нечасто говорит.

>>72490
PBS SpaceTime про космос и астрофизику. А вот именно про ракеты щас почему-то не упомню, у кого-то вроде видел серию. Но того же Скотта навернуть стоит, толково вещает.
watman.png277 Кб, 352x476
493 372536
>>72535

>disable_polymer



Чегойта так мало видео у него.
494 372538
>>72501
>>72535
>>72536
Благодарю, ананасы!
495 372583
>>72534
Основы орбитальной механики и про всякие эффекты Обертов у него можно узнать в кспшных видосиках, а про ирл ракеты у него тоже прилично контента но хотелось бы больше(
изображение.png754 Кб, 900x423
496 372618
Правда ли, что межзвездные корабли будущего будут скорей всего простыми шарами, а не йобами как на пике?
Bsphere.jpg1 Мб, 900x853
497 372620
>>72618
Разумеется, нет. Я прибыл из 2277 и ответственно заявляю, что шары только у Боргов, остальные корабли имеют интересные формы.
Сдурел что ли, откуда мы можем будущее знать? Не факт еще, что человечество сумеет в межзвездные полеты.
498 372636
>>72618
Вряд ли, если конечно мы говорим об обозримом будущем. Сферический корабль сложно собирать, сложно защищать от набигающих частиц, и если на него биоматериал грузить, то придётся делать искусственную гравитацию. Так что примерно как на твоём пике, но только у тебя скорее межпланетный, а у межзвёздного будет ещё прицеплен к этому вагон раз в 10 больше этого, заполненный топливом и водой.
499 372639
>>72618
Нет, делать сферой имеет смысл только обитаемый отсек. На пике 1 один из вариантов корабля будущего с ЯРД (только такие иллюминаторы не нужны) - обитаемый отсек в виде сферы, сопло ЯРД максимально далеко разнесено от жилого отсека и экранировано радиационным щитом, радиаторы - в "тени" радиационного щита, дополнительное экранирование экипажа за счет топливных баков. Пик 2 - ЭЯРД, как вариант. Кто-то вообще планирует реактор на тросе длиной в километр буксировать, чтобы не приходилось утяжелять корабль экранированием.
500 372640
>>72618
Нет, межзвёздные корабли, скорее всего, будут иметь форму прямоугольного параллелепипеда.
https://antihydrogen.livejournal.com/46578.html
Это если без пузыря альбукерке.
501 372641
>>72639
Нахуя обитаемый отсек в виде сферы? Преимуществ никаких, модульности нет, искусственной гравитации нет. И на твоих картинках нет никакой защиты от межзвёздной среды.
502 372642
>>72163

>Часы


Это прост максимально стабильный генератор частоты, о величине и методе генерации которой все договорились
503 372643
>>72641
потому что это высер фантастов голливудских.
который к реальности отношения вообще не имеет.
>>72618
да это так.
504 372645
>>72643

>да это так.


Назови хоть одну причину, почему это должно быть так.
505 372647
>>72639

>На пике 1 один из вариантов корабля будущего с ЯРД


Совсем на своём сайфаче ёбнулся? Это моделька по мотивам Космической Одиссеи 2001 от какого-то аутиста:
http://theminiaturespage.com/boards/msg.mv?id=335076
506 372654
Поясните, так сколько человек в невесомости пробыл? Почему-то в одной статье читаю что пара дней и человеку уже очень плохо, а месяц почти никто не продерживался. А в другой читаю что чуть ли не годами на мкс живут.
65464262345141.jpg9 Кб, 311x162
507 372655
>>72645
по логике которую ты все равно своим мозжечком не поймешь
508 372656
>>72654
Ты в курсе, что люди разные? Некоторых на океанском лайнере укачивает сразу на трапе, некоторые годами океаны бороздят на ветхой шхуне. Так и тут - у некоторых обнаруживается космическая болезнь, притом не сразу порой, а некоторые годы наматывают и становятся легендами.
509 372657
>>72654
Потому что методы разные. Одно дело в консервенной банке "Союза" проболтаться две недели, и другое - на МКС, где куча тренажеров, на которых космонавты по нескольку часов в день впахивают, чтобы держать себя в форме. Через пару дней "плохо" это чисто субъективные ощущения от охуевающего вестибулярного аппарата, проходит через недельку обычно.
Так - вон недавно Келли с Корниенко год на орбите пробыли, Поляков больше 400 дней.
510 372658
>>72657
А что в статье на гиктаймс пишут что неизвестно как люди будут вести себя месяцами при перелёте на марс в замкнутом пространстве без гравитации, если уже есть такие данные?
511 372659
>>72618

Ну к времени межзвёздных перелётов скорее всего звездолётчики пройдут процедуру биотрансференции. А межпланетные корабли будут иметь всякие торчащие хуёвины, вроде солнцезащитных экранов, солнечных батарей, антенн и прочей хуеты. Как МКС, Вояджер или другие КА. Шар, а зачем шар? Я не знаю зачем шар.
В 2250-м году на Земле будет жить хуева гора всякого скама, вроде русни, каказцев, какеллов, нигер, пиндосов, хуемразей, двачеров и негров, которые будут только дрочить и капчевать, а работать не будут. Так как хранить их негде, а убивать негуманно, то их будут погружать в виртуальную реальность аля пелевинская "Любовь к трём цукербринам", затем паковать в консервные банки и отправлять нахуй, то есть колонизировать неведомые ебеня. Так как проблема биологического старения будет решена, то межзвёздный перелёт перестанет пугать сроками. Прибыв на новую планету оккупационная межзвёздная баржа "спейсач" высадит наноботов и спейсачемарин, где сходу начнёт строить битардск и двачеформировать планету.
512 372660
>>72658
Так лететь пока хотят в говнокапсулах, там непонятно.
Если йоба-корабль размерами с МКС будет, то проблем не будет.
Но всё равно не полетят, так что кукареканья в гиктаймсах ими и останутся.
513 372661
>>72660
Инфа что не полетят, и почему?
514 372663
>>72658
Либо учёный изнасиловал журналиста, либо ты не так понял. И закрытое пространство уже изучено что на Мире/МКС, что в куче наземных инструментов, и невесомость. Скорее всего, первое время на Марсе в скафандрах будет немножко хреново, потому что полностью компенсировать влияние невесомости на организм пока не получается, без скафандров будет норм. Со временем адаптируются.
515 372664
>>72661
А с чего бы им лететь-то? И кто запускать будет?
Даже СЛС не потянет, они только на окололунную слетают (даже посадки не планируется), а это самая йобистая ракета что будет.
516 372665
>>72658
Давно в очи сношаешься?

>Келли с Корниенко год на орбите пробыли, Поляков больше 400 дней.


Полёт до Масра - 200-300 дней, Маринер-9 вообще за 168 долетел.
>>72660

>Так лететь пока хотят в говнокапсулах, там непонятно.


Пока не хотят лететь, а только рендеры рисуют.
В любом случае, в капсуле типа Ориона никого не запустят, в таком полёте кто угодно ебанётся на отличненько. Если что будут делать жилой модуль - это как минимум.
517 372666
>>72664

>Даже СЛС не потянет, они только на окололунную слетают


Ты из секты Свидетелей Однопуска, или из секты Отрицателей Разгонных Блоков?
518 372667
>>72665

>В любом случае, в капсуле типа Ориона никого не запустят, в таком полёте кто угодно ебанётся на отличненько.



Можно взять заранее ёбнутого двачера, с вармартыхача или ещё откуда. Он и не заметит ничего, да и не жалко его особо.
519 372668
>>72665

>>Так лететь пока хотят в говнокапсулах, там непонятно.


>Пока не хотят лететь, а только рендеры рисуют.


Давно в очи сношаешься? >>72660

>Но всё равно не полетят



>>72666
Я не из секты. Просто манякартинок можно нарисовать миллиард, а на деле смотреть надо на то, что делается. СЛС делается и будет 100%. Другие сверхтяжи не делаются в той степени чтобы смело говорить что они будут 100% летать.
Илон-то уже три раза свою пилотируемую концепцию менял, например. Что с российскими заменами Союза так я вообще помолчу.
520 372670
Когда я читаю про всякие супер-йобы типа итэр или уэбб, мне кажется что что-то пойдёт не так и она йобнет.
521 372671
>>72670
ИТЭР не ёбнет, это просто толстый токамак, их сотни и работают те больше полувека.
А если Уэбб упадёт будет крайне обидно, то ракета, то разгонный блок, то собственные системы ориентации как у Кеплера отказывают регулярно.
522 372672
>>72671
УЭбб на сверхнадёжной Ариан-5, хоть это обнадёживает.
523 372673
>>72672
Дискавери тоже сверхнадёжными считались.
524 372675
>>72668

>Просто манякартинок можно нарисовать миллиард, а на деле смотреть надо на то, что делается. СЛС делается и будет 100%. Другие сверхтяжи не делаются в той степени чтобы смело говорить что они будут 100% летать.


Я доебался говорю не про то.

>СЛС не потянет


Я правильно понимаю, что с помощью СЛС нельзя собрать на орбите хуиту и вывести к ней разгонный блок, который это всё отправит к Марсу? Какой из законов физики этому воспрепятствует?
525 372676
>>72673
Ээ, кто?
526 372678
>>72676
Или колумбия, забыл как назывался американский шаттл.
main900.jpg72 Кб, 900x596
527 372679
>>72673
Дискавери и есть сверхнадёжный - 27 лет эксплуатации, 39 успешных полётов. Что не так?
528 372680
>>72678
Американский шаттл назывался Шаттл. Сверхнадёжными они считались на бумаге, а Ариан-5 доказал надёжность делом - одна авария, и та в самом первом полёте.
529 372681
>>72675
Можно собрать хуиту.
Но планов на это нет. Это ж НАСА, она не импровизирует. Еще непонятно, будут ли ARM выполнять.
Так-то подумать, никто не мешал то же самое что ты предлагаешь на Сатурн-5 делать.
530 372682
>>72681
Да мне похуй на планы, и так всё ясно. Я просто подумал что ты ебанутый, а ты не ебанутый.
531 372696
>>72655
Ну понятно, значит тебе лишь бы спиздануть абы что.
532 372707
>>72641

>Нахуя обитаемый отсек в виде сферы? Преимуществ никаких, модульности нет, искусственной гравитации нет. И на твоих картинках нет никакой защиты от межзвёздной среды.


Минимальный размер внешней оболочки по сравнению с внутренним объемом.

> межзвёздной среды


Ты хотел сказать космическое излучение? Потому что это дальше Солнечной системы не улетит. Буду признателен, если так же научишь по картинкам громогласно заявлять, есть ли там защита или нет.

>>72643
>>72647
Очень взвешенная и трезвая аргументация, просто сражен наповал. Раз имеет отношение к научной фантастики (как будто это что-то плохое), значит априори говно. Расскажите что не так, а еще лучше покажите на примеры получше с учетом технологий недалёкого будущего, без фотонных двигателей и прочих манек.
533 372710
>>72707

>Минимальный размер внешней оболочки по сравнению с внутренним объемом.


Да похуй на это вообще. С оболочкой и так проблем никаких, а вот в остальном сфера нахуй не нужна.

>Ты хотел сказать космическое излучение?


Нет, я хотел сказать межзвёздная среда. И сказал.

>Буду признателен, если так же научишь по картинкам громогласно заявлять, есть ли там защита или нет.


Ну смотри. Видишь вот первую картинку здесь >>72639, в которой кабина из Космической Одиссеи? Ну вот на ней видно, что стенки кабины от силы полметра толщиной, и больше ничего впереди нету. Это недостаточная защита. На втором пике хуй знает, там вообще не разглядеть ничего.
GuGk4m3.jpg518 Кб, 1920x1311
534 372713
>>72707

>Раз имеет отношение к научной фантастики, значит априори говно.


Вот у меня тоже тут вариант корабля будущего, базарю, такие и будут.
А теперь перестань называть поделки скучающих моделистов вариантами кораблей будущего, у меня тут настоящие чертежи хотя бы. Ты же не будешь в разделе про космос пренебрежительно относиться к такой вещи, как репульсорные двигатели, использующие субатомные узлы пространства-времени, получающиеся на автоматических заводах в окрестностях чёрных дыр, потому что Лукас вынул их из жопы пару умных слов чтобы круто звучало?
Без имени-1.jpg664 Кб, 3327x2000
535 372714
>>72707
Для баков плюс, для жилых модулей - не.
536 372715
>>72713

Твой пик к научной фантастике имеет такое же отношение, как Рогозин к отечественным космическим успехам.

>>72710

С чего ты взял, что экипаж нужно защищать от радиации?
537 372716
>>72715
В сортах выдуманных режиссёрами корабликов не разбираюсь, извини.
538 372721
>>72716

Нет, не извиню.
Зачем в космическом корабле такого размера турбины, например? Если ты не отличаешь космооперу от научной фантастики, то сходи и разберись. Научной, именно научной, фантастики мало. Катастрофически мало.
539 372747
>>72715

>С чего ты взял, что экипаж нужно защищать от радиации?


Во-первых, с чего ты взял, что не нужно? Во-вторых, не о радиации речь, в смысле, не об излучении солнца. Хотя и от него нужно защищать, и от излучения звезды, к которой летим. Но это всё хуйня, от этого защититься несложно. Я говорю о частицах межзвёздной среды, которые с релятивистскими скоростями будут налетать на твой релятивистский звездолёт. Третий блядь раз пишу.
540 372749
>>72721
Какие турбины, где?
541 372763
>>72749

На этой пикче >>72713 в кормовой части main engine turbine, около центрального сопла.

>>72747

Понял тебя.
К тому времени, когда человечество сможет в межзвёздные перелёты, то оно сможет модифицировать свои тела настолько, что устойчивость к радиации вырастет на несколько порядков. Об этом писал Уоттс, например, а он в биологии шарит.

>Я говорю о частицах межзвёздной среды



Понял тебя. Я

>релятивистский звездолёт.



Для релятивистского звездалёта оптические фотоны станут источниками радиации, за счёт фиолетового смещения. Я думал ты говоришь про совсем релятивистский случай, когда реликтовое излучение в гамму превращается.
542 372767
>>72763

>К тому времени, когда человечество сможет в межзвёздные перелёты, то оно сможет модифицировать свои тела настолько, что устойчивость к радиации вырастет на несколько порядков.


Тыскозал? Или Уоттс? Откуда ему блядь знать, когда человечество сможет в межзвёздные перелёты, и что оно к этому времени сможет модифицировать? В 2100 году сможет? Хуй блядь знает. Кроме того, защищать не только людей надо будет, а и всю электронику, и всё вообще.

>Для релятивистского звездалёта оптические фотоны станут источниками радиации, за счёт фиолетового смещения.


Ну ок, станут.
543 372769
>>72767

>Уоттс?



Да.

>Откуда ему блядь знать, когда человечество сможет в межзвёздные перелёты, и что оно к этому времени сможет модифицировать? В 2100 году сможет?



Ну в книжке в 2082-м году корабль зашвырнули к облаку Оорта, с рептилоидами инопланетянами разбираться. Экипаж в консервированно-сушёном виде летел. На барже инопланетян была сильная радиация, электроника её переносила хуже, чем экипаж. Вообще довольно неплохо написано, поскольку межзвёздный перелёт требует либо бессмертного экипажа, либо искривлятора пространства. Династиями лететь так себе занятие.
544 372780
>>72710

> Нет, я хотел сказать межзвёздная среда.


Зачем защита от межзвездной среды кораблю с УИ двигателя в 800-4000 cекунд?

>>72713

> А теперь перестань называть поделки скучающих моделистов вариантами кораблей будущего, у меня тут настоящие чертежи хотя бы


А чьи поделки можно называть вариантами корабля будущего по твоему мнению?

>>72714

> картинка уровня /b


Посмотри как нибудь на досуге, какой формы поперечное сечение у орбитальных модулей космических станций, для начала. Везде идиоты, сколько места пропадает же.
545 372790
>>72780
Все модули – цилиндры, потом что там только один "этаж". Если пускать йобу с несколькими палубами, то цилиндр, а уж тем более сфера ебаная невыгодна
skylab.jpg90 Кб, 415x540
546 372792
>>72790

Нутупые!!!
547 372793
>>72769

>Да.


Это кстати не тот, у которого в одном рассказе люди не эволюционировали нихуя за миллиард лет на земляшке в отсутствие цивилизации? В любом случае, так себе аргумент. Никто сейчас не может такие технологии предсказывать.

>Экипаж в консервированно-сушёном виде летел


Ну это вообще ионная пушка. Такая хуйня может появиться через 50 лет, а может (и, мне кажется, вероятнее) через 150, а то и больше. Вообще хрен предскажешь.

>межзвёздный перелёт требует либо бессмертного экипажа, либо искривлятора пространства


Да нет, не требует. Летишь себе и ладно. Твои внуки прилетят.

>Династиями лететь так себе занятие.


Куча людей живут где-то в ебенях в деревне всю жизнь, занимаются своим трудом и не жалуются. Тут не хуже. Уверен, что если бы прямо сейчас предложили полететь в систему, где вероятно есть жизнь и сносные условия для людей, и полёт бы занял 200 лет, то нашлись бы сотни, если не тысячи добровольцев.

>>72780

>Зачем защита от межзвездной среды кораблю с УИ двигателя в 800-4000 cекунд?


Я не очень понимаю, при чём тут это, но ты молодец, что умеешь предсказывать УИ двигателей межзвёздных кораблей неизвестной конструкции.
548 372795
>>72793

>Это кстати не тот, у которого в одном рассказе люди не эволюционировали нихуя за миллиард лет на земляшке в отсутствие цивилизации?



Это тот, у которого я лазил в википедия в среднем один раз за полторы страницы.

>Такая хуйня может появиться через 50 лет



Ну да, там это была вынужденная мера а на дворе был 2082-й год. В любом случае это проще, чем релятивистские космобли.

>Летишь себе и ладно. Твои внуки прилетят.


>Уверен, что если бы прямо сейчас предложили полететь в систему, где вероятно есть жизнь и сносные условия для людей, и полёт бы занял 200 лет, то нашлись бы сотни, если не тысячи добровольцев.



Сейчас есть сотни и тысячи добровольцев слетать на Марс сильно рискуя, да ещё и за свои деньги. Дело не только в добровольцах.

>>72793

>но ты молодец, что умеешь предсказывать УИ двигателей межзвёздных кораблей неизвестной конструкции.



Я не он, но если ты говоришь про 50 и 150 лет, то маловероятно что будет создано что-то превосходящее ионник по УИ. Напомни мне, как вырос УИ за последние 50 лет?
549 372800
>>72795

>Я не он, но если ты говоришь про 50 и 150 лет, то маловероятно что будет создано что-то превосходящее ионник по УИ


https://en.m.wikipedia.org/wiki/Dual-Stage_4-Grid
Вот тебе ионник. Да даже у тех, что уже сейчас работают 10000 есть.

>Напомни мне, как вырос УИ за последние 50 лет?


Нехило вырос.
14920934239990.jpg1,1 Мб, 2160x2160
550 372816
В СС есть планеты-гигант - Уран, Нептун, Сатурн и Юпитер.
И чем глубже мы опускаемся в них от условного уровня поверхности атмосферы, тем плотнее и жёстче условия по имеющимся ныне данным. И водород где-то там металлический, и кислород там красный кристаллический. Так вот, а можно ли за счёт ЕСЛИ прилететь на Уран, повыкапывать с киркой и очень плотных слоёв толи ещё атмосферы, толи уже океана, толи уже суши кристаллы металлического красного кислорода, гробо говоря, пораспихать кислородные кристаллы в карманы, и вернуться к себе домо на Земляшку поставить уранский кислород на полочку сувениром, будет ли этот кислород оставать красным металлическим кристаллом кислорода, или попав на Землю раскукожится на обычный атомы и молекулы кислорода?
551 372817
>>72816

>Землю раскукожится на обычный атомы и молекулы кислорода


This
552 372818
>>72816
Поговаривают, что тот металлический водород, который прессовали алмазами, является мета-стабильным и его не раскукоживает.
Lexx Ship.jpg1,1 Мб, 4000x2500
553 372820
>>72618
>>72639

Нет, делать сферой имеет смысл только обитаемый отсек. На пике 1 один из вариантов корабля будущего с ЯРД (только такие иллюминаторы не нужны) - обитаемый отсек в виде пары сфер, сопло ЯРД максимально далеко разнесено от жилого отсека и экранировано радиационным щитом, радиаторы - в "тени" радиационного щита, дополнительное экранирование экипажа за счет топливных баков.
14575531073453.gif777 Кб, 500x500
554 372821
>>72818
Это тот самый, который они потеряли, уронив на пол?
555 372822
>>72707

>Минимальный размер внешней оболочки по сравнению с внутренним объемом.


Лукьяненко обчитался? Давайка пиздуй в sf.
worst post ever.png419 Кб, 803x688
556 372823
>>72822
Проиграл. Когда у вас в школе стереометрия будет - поймёшь.
557 372824
>>72821
Насчет этого не слышал. Но там вроде говорилось, что он остается в режиме металлического даже после расжатия алмазных наковален.
558 372825
>>72823
Скажи быдло тупое, чем же так важен минимальный размер внешней оболочки?
559 372826
>>72824

>он остается в режиме металлического даже после расжатия алмазных наковален


Вот только видел этот водород только один ученый-шарлатан и всего пару секунд.
image.png339 Кб, 604x579
560 372827
>>72825
МАССОЙ.
561 372828
>>72826

>ученый-шарлатан


Начинается.
562 372842
>>72800

>Нехило вырос.



Цифры-циферки есть, или у нас гуманитарная космонавтика?

>Вот тебе ионник. Да даже у тех, что уже сейчас работают 10000 есть.



Думаю ты сам понимаешь какие недостатки у ионников.
563 372845
>>72827
По твоему толшина внешних стенок весит сотни тонн?
564 372847
>>72845
Началось, блядь.
Сфера имеет наибольший объем при наименьшей площади поверхности среди всех геометрических фигур. Всё. Это простая стереометрия. Всякие писаки тут ни при чём. Всякие особенности вымышленных кораблей ни при чём.
Зачем-то отвечая на вопрос: В ракетной отрасли буквально каждый грамм на счету.
OriondockedtoMarsTransferVehicle-1.jpg237 Кб, 1600x900
565 372856
>>72790

> Все модули – цилиндры, потом что там только один "этаж". Если пускать йобу с несколькими палубами, то цилиндр, а уж тем более сфера ебаная невыгодна


Тебе не приходило в голову, что при проектировании любое конструкторское решение имеет плюсы и минусы? Сфера обеспечивает максимальный объем при минимальной площади поверхности, а за счет отсутствия напряжения на рёбрах, которых у сферы просто нет, можно сделать толщину стенок корабля потоньше, сократив массу корабля до минимума, и соответственно взяв больше полезного груза. Опять же если нужна защита от того же ионизирующего излучения, минимальная площадь - минимальная масса защиты.

Цилиндры (которые в поперечном сечении представь себе - круг!) для модулей станций и кораблей используют вместо сферы по одной простой причине, что этот модуль или корабль еще как-то надо доставить на орбиту, а это накладывает ограничения на максимальный диаметр модуля, поэтому увеличивают длину. Кстати, если ты обратишь чуть больше внимания, заметишь, что у этих "цилиндров" очень часто округлены грани, чтобы этого напряжения избежать и не пришлось чтобы утяжелять конструкцию её усилением.

Именно по этой причине корабли будущего (по крайней мере до тех пор, пока не изобретут ракетные двигатели, где тяга будет идти напрямую за счет реакции деления/синтеза ядер) будут с сигарообразными или шарообразными герметизированными модулями, потому что каждый грамм на счету.

Почему же скорее шар, а не цилиндр теперь? Полагаю, что корабль будущего будут напрямую конструировать на орбите, поэтому не надо будет скукоживать модуль по продольной оси.

>>72793

> Я не очень понимаю, при чём тут это, но ты молодец, что умеешь предсказывать УИ двигателей межзвёздных кораблей неизвестной конструкции.


Объясняю - та моделька Дискавери или ионника - это корабль хоть и будущего, но по части технологического уровня его можно было еще в 70-х начать проектировать. Защита от межзвездной среды для такого корабля всё равно что спойлер для Трактора Беларусь, не то что не нужна, а абсурдна сама такая идея. И если это было непонятно после тонких намёков, дальше можешь не продолжать.
OriondockedtoMarsTransferVehicle-1.jpg237 Кб, 1600x900
565 372856
>>72790

> Все модули – цилиндры, потом что там только один "этаж". Если пускать йобу с несколькими палубами, то цилиндр, а уж тем более сфера ебаная невыгодна


Тебе не приходило в голову, что при проектировании любое конструкторское решение имеет плюсы и минусы? Сфера обеспечивает максимальный объем при минимальной площади поверхности, а за счет отсутствия напряжения на рёбрах, которых у сферы просто нет, можно сделать толщину стенок корабля потоньше, сократив массу корабля до минимума, и соответственно взяв больше полезного груза. Опять же если нужна защита от того же ионизирующего излучения, минимальная площадь - минимальная масса защиты.

Цилиндры (которые в поперечном сечении представь себе - круг!) для модулей станций и кораблей используют вместо сферы по одной простой причине, что этот модуль или корабль еще как-то надо доставить на орбиту, а это накладывает ограничения на максимальный диаметр модуля, поэтому увеличивают длину. Кстати, если ты обратишь чуть больше внимания, заметишь, что у этих "цилиндров" очень часто округлены грани, чтобы этого напряжения избежать и не пришлось чтобы утяжелять конструкцию её усилением.

Именно по этой причине корабли будущего (по крайней мере до тех пор, пока не изобретут ракетные двигатели, где тяга будет идти напрямую за счет реакции деления/синтеза ядер) будут с сигарообразными или шарообразными герметизированными модулями, потому что каждый грамм на счету.

Почему же скорее шар, а не цилиндр теперь? Полагаю, что корабль будущего будут напрямую конструировать на орбите, поэтому не надо будет скукоживать модуль по продольной оси.

>>72793

> Я не очень понимаю, при чём тут это, но ты молодец, что умеешь предсказывать УИ двигателей межзвёздных кораблей неизвестной конструкции.


Объясняю - та моделька Дискавери или ионника - это корабль хоть и будущего, но по части технологического уровня его можно было еще в 70-х начать проектировать. Защита от межзвездной среды для такого корабля всё равно что спойлер для Трактора Беларусь, не то что не нужна, а абсурдна сама такая идея. И если это было непонятно после тонких намёков, дальше можешь не продолжать.
566 372891
>>72847
Судя по желтому колобку рака тебе и капсболду краба тебе, там вес идет на тонны, а не на каждый грамм.
567 372973
>>72659
Что-то я сомневаюсь, что проблему биологического старения удастся решить. Бога не обмануть.
568 372975
>>72721
Александр Воробьев и его "Огненный след" считаются НАУЧНОЙ фантастикой? Его корабли реалистичны?
14794807712962.gif828 Кб, 500x300
569 372976
Человек - разумная форма жизни
Гриб - неразумная форма жизни
Робот с ИИ - разумная форма нежизни
А в чём критерий жизни и нежизни и разума/неразума?
Вот прилетает человечество в систему другой звезды и обнаруживает там какие-нибудь камни или жидкости, явно перемещающиеся по своей воле, но не являющиеся ни ивым ни при каких обстоятельствах, ни проявляют признаков разума, хоть и абсолютно точно не является неразумным. Вот как тогда такое классифицировать? не разумное и не неразумное, не живое и не неживое.
570 372987
Каков процент систем с числом звёзд 3 и более, от общего числа систем?
571 372989
>>72987
Кратная звезда
[править | править код]
Материал из Википедии — свободной энциклопедии

Тройная звезда HD 188753
Кратная звезда состоит из трёх или более звёзд, которые выглядят с Земли близкими друг к другу. Эта близость может быть просто видимостью (звезды, расположенные на разных расстояниях, находятся близко по лучу зрения) — в этом случае звезда называется оптически кратной, или является следствием того, что звёзды находятся физически близко и связаны друг с другом гравитацией — в этом случае звезда называется физически кратной. Физически кратные звёзды — это разновидность кратной звёздной системы.

Если звёзды — компоненты физически кратной системы могут быть разрешены (то есть их можно увидеть по отдельности в телескоп), такая система называется визуально кратной. Если же кратность звезды может быть определена только с помощью спектральных (доплеровских) или фотометрических (по изменению блеска) наблюдений, она называется спектрально кратной или затменной кратной системой.

Существуют системы с большими кратностями (например, система Кастора состоит из 6 компонентов[1]). При этом в звёздную систему максимально может входить не более 10 звёзд[источник не указан 1581 день]. При их большем числе считается, что это звёздное скопление — более крупная единица в астрономии.

Примеры[править | править код]
HD 188753 — физически кратная звезда с тремя компонентами: HD 188753 A (жёлтый карлик), HD 188753 B (оранжевый карлик) и HD 188753 C (красный карлик). Звёзды B и C обращаются вокруг друг друга за 156 дней, и, совместно, вокруг A за 25,7 лет.
HR 3617 — кратная звезда с тремя компонентами, HR 3617 A, HR 3617 B и HR 3617 C. A и B образуют физически двойную звезду, а C — оптически кратную.
Глизе 570 — ближайшая к Солнцу кратная звезда с четырьмя компонентами (19 св. лет), состоящая из двух красных, одного оранжевого и одного коричневого карликов.
ε Лиры — кратная звезда с четырьмя компонентами. Система состоит из двух двойных звезд.
BD−22°5866 — кратная звезда с четырьмя компонентами.
Kepler-64 — кратная звезда с четырьмя компонентами.
30 Овна — кратная звезда с четырьмя компонентами.
Кастор - шестикратная звезда.
571 372989
>>72987
Кратная звезда
[править | править код]
Материал из Википедии — свободной энциклопедии

Тройная звезда HD 188753
Кратная звезда состоит из трёх или более звёзд, которые выглядят с Земли близкими друг к другу. Эта близость может быть просто видимостью (звезды, расположенные на разных расстояниях, находятся близко по лучу зрения) — в этом случае звезда называется оптически кратной, или является следствием того, что звёзды находятся физически близко и связаны друг с другом гравитацией — в этом случае звезда называется физически кратной. Физически кратные звёзды — это разновидность кратной звёздной системы.

Если звёзды — компоненты физически кратной системы могут быть разрешены (то есть их можно увидеть по отдельности в телескоп), такая система называется визуально кратной. Если же кратность звезды может быть определена только с помощью спектральных (доплеровских) или фотометрических (по изменению блеска) наблюдений, она называется спектрально кратной или затменной кратной системой.

Существуют системы с большими кратностями (например, система Кастора состоит из 6 компонентов[1]). При этом в звёздную систему максимально может входить не более 10 звёзд[источник не указан 1581 день]. При их большем числе считается, что это звёздное скопление — более крупная единица в астрономии.

Примеры[править | править код]
HD 188753 — физически кратная звезда с тремя компонентами: HD 188753 A (жёлтый карлик), HD 188753 B (оранжевый карлик) и HD 188753 C (красный карлик). Звёзды B и C обращаются вокруг друг друга за 156 дней, и, совместно, вокруг A за 25,7 лет.
HR 3617 — кратная звезда с тремя компонентами, HR 3617 A, HR 3617 B и HR 3617 C. A и B образуют физически двойную звезду, а C — оптически кратную.
Глизе 570 — ближайшая к Солнцу кратная звезда с четырьмя компонентами (19 св. лет), состоящая из двух красных, одного оранжевого и одного коричневого карликов.
ε Лиры — кратная звезда с четырьмя компонентами. Система состоит из двух двойных звезд.
BD−22°5866 — кратная звезда с четырьмя компонентами.
Kepler-64 — кратная звезда с четырьмя компонентами.
30 Овна — кратная звезда с четырьмя компонентами.
Кастор - шестикратная звезда.
572 372994
>>72975
Взглянул мельком на текст:

> вирт-привод


> виртуалный тоннель


Нет.

По виду - морской бой с измененными декорациями, технологии есть, но без серьезного углубление, главное создать антураж и возможность осуществления тех или иных действий во внутреннем мире книги. Типичные технологии на анобтаниуме.

> Семь десятых «же» — максимум, что мог выдать двигатель нагруженного транспорта в форсированном режиме. Крауз как наяву увидел многокилометровый хвост ионизированного водорода, тянущийся словно на привязи за кормой разгоняющегося корабля. Со стороны факел смотрится чертовски красиво! Эдакий поток слепящего огня, который постепенно расширяется, тускнеет и сходит на нет. — Капитан! Запас рабочего тела всего на четырнадцать часов форсированной тяги, — предупредил навигатор, отрываясь от своего рабочего монитора.


У меня нет расчетов совершаемой работы, но что-то мне подсказывает, что для такой производительности нужна тяга из термоядерной реакции, а не ионизированный водород. Вообще очень странный выбор рабочего тела, учитывая характеристики водорода.
573 372997
>>72973

>Бога не обмануть.



Бога нет.

>>72975

>Жанр:боевик, фантастика, космическая фантастика


>Далекое будущее. Все страны объединились под эгидой «Лига». Ближайшие планеты колонизированы. Человечество ведет размеренную жизнь. Неожиданно появляется новая высокоразвитая раса. Первый контакт оборачивается долгой и страшной войной. Инопланетяне имеют мощнейший флот, сильное вооружение. Намерения их жестоки. Человечество принимает войну, которая обещает уничтожением одной из рас. Ставки слишком высоки, двум цивилизациям не ужиться в космосе. Денис Демин — простой пилот танкшипа, оказывается в горниле масштабного космического сражения, где решается судьба всего человечества



Пфффффф, нет конечно.
574 372998
>>72989
То есть из 200-400 миллиардов звёзд в галактике только 10 не являются двойными или одинарными?
313.jpg105 Кб, 1024x681
575 373040
С этим вопросом лучше обращаться к прекрасной половине человечества, но вряд ли таковая представлена в /spc/.
Итак, что происходит у женщин с грудью в невесомости? Через нее равномерно перекачивается кровь всего организма? Представляет ли это какую-либо опасность для здоровья в дальнейшем?
P.S. Серьезно, не тролль. Действительно интересно.
576 373049
>>72997
Анусдушу ставишь?
577 373055
>>72998
Ориентировочно. На деле более половины систем двойные и кратные, довйных большинство, менее половины - одиночные.
578 373057
>>73040
Почему к женщинам вопрос? Думаешь, дамы не бывавшие в невесомости знают всё о своих органах во всех ситуациях?
С сиськами то же самое, что и со всеми другими органами - кровь прокачивается точно так же, даже чуть больше из-за того, что те более не находятся выше центра тела (как и голова).
Никакой опасности из-за микрогравитации точно так же, как и с пенисом и другими торчащими частями тела.
И плюс - лифчик не надо носить, поддерживать сиськи незачем без притяжения.
579 373063
>>73057

>И плюс - лифчик не надо носить, поддерживать сиськи незачем без притяжения.


То-то Саманта его и не носит нам на радость.
580 373064
>>73063
То-то с прилетом Саманты на борт мочеотсос в сортире стал забиваться совсем не мочой.

Я сам себя выпровожу.
581 373073
На сколько градусов теплее реликтового излучения там где мы щас из-за окружающих звезд и галактик если не считать влияния Солнца?
582 373087
>>73064
А чем?
583 373088
>>73087
Мочой.
Хотел пошутить про малафью, но потом понял, как это тупо и пожалел про тот пост.
584 373090
Хм... !
Ведь с открытием гравитационных волн, скоро можно будет построить много радаров в разных точках и с очень высокой точностью определять местоположение объекта излучения гарвитационных волн, а по полученным данным уже искать эти объекты и снимать их в разных спектрах!
585 373092
>>73090
Уже.
586 373114
>>73090
Слияние нейтронок же точно сдетектили?
587 373157
>>73090
Ты 2017 год в коме провел что ли?
https://en.wikipedia.org/wiki/GW170817
588 373167
Этому треду не помешает лишний перекат.
ПЕРЕКОТ 589 373254
590 373280
>>73167
>>73254
Ай блжад, я вчера еще хотел запилить.
Уже второй тред не я ОП. Аж грустно немного.
591 373386
>>72976

>по своей воле


>ни проявляют признаков разума

Тред утонул или удален.
Это копия, сохраненная 12 сентября 2018 года.

Скачать тред: только с превью, с превью и прикрепленными файлами.
Второй вариант может долго скачиваться. Файлы будут только в живых или недавно утонувших тредах. Подробнее

Если вам полезен архив М.Двача, пожертвуйте на оплату сервера.
« /spc/В начало тредаВеб-версияНастройки
/a//b//mu//s//vg/Все доски